How to prove that exponential grows faster than polynomial?












56












$begingroup$


In other words, how to prove:




For all real constants $a$ and $b$ such that $a > 1$,



$$lim_{nrightarrowinfty}frac{n^b}{a^n} = 0$$




I know the definition of limit but I feel that it's not enough to prove this theorem.










share|cite|improve this question











$endgroup$








  • 5




    $begingroup$
    Have you tried expressing both in terms of the exponential?
    $endgroup$
    – gary
    Aug 4 '11 at 1:41










  • $begingroup$
    You could write the exponential as an infinite polynomial power series. You could also see what happens when you take higher and higher order derivatives of $n^b$ and $a^n$: the polynomial vanishes by an order each time and the exponential is only multiplied by a constant factors ($ln a$) each time.
    $endgroup$
    – jnm2
    Aug 4 '11 at 2:36
















56












$begingroup$


In other words, how to prove:




For all real constants $a$ and $b$ such that $a > 1$,



$$lim_{nrightarrowinfty}frac{n^b}{a^n} = 0$$




I know the definition of limit but I feel that it's not enough to prove this theorem.










share|cite|improve this question











$endgroup$








  • 5




    $begingroup$
    Have you tried expressing both in terms of the exponential?
    $endgroup$
    – gary
    Aug 4 '11 at 1:41










  • $begingroup$
    You could write the exponential as an infinite polynomial power series. You could also see what happens when you take higher and higher order derivatives of $n^b$ and $a^n$: the polynomial vanishes by an order each time and the exponential is only multiplied by a constant factors ($ln a$) each time.
    $endgroup$
    – jnm2
    Aug 4 '11 at 2:36














56












56








56


35



$begingroup$


In other words, how to prove:




For all real constants $a$ and $b$ such that $a > 1$,



$$lim_{nrightarrowinfty}frac{n^b}{a^n} = 0$$




I know the definition of limit but I feel that it's not enough to prove this theorem.










share|cite|improve this question











$endgroup$




In other words, how to prove:




For all real constants $a$ and $b$ such that $a > 1$,



$$lim_{nrightarrowinfty}frac{n^b}{a^n} = 0$$




I know the definition of limit but I feel that it's not enough to prove this theorem.







calculus limits exponential-function






share|cite|improve this question















share|cite|improve this question













share|cite|improve this question




share|cite|improve this question








edited Jun 21 '18 at 18:21









Wouter

5,96921436




5,96921436










asked Aug 4 '11 at 1:33









ablmfablmf

2,58352452




2,58352452








  • 5




    $begingroup$
    Have you tried expressing both in terms of the exponential?
    $endgroup$
    – gary
    Aug 4 '11 at 1:41










  • $begingroup$
    You could write the exponential as an infinite polynomial power series. You could also see what happens when you take higher and higher order derivatives of $n^b$ and $a^n$: the polynomial vanishes by an order each time and the exponential is only multiplied by a constant factors ($ln a$) each time.
    $endgroup$
    – jnm2
    Aug 4 '11 at 2:36














  • 5




    $begingroup$
    Have you tried expressing both in terms of the exponential?
    $endgroup$
    – gary
    Aug 4 '11 at 1:41










  • $begingroup$
    You could write the exponential as an infinite polynomial power series. You could also see what happens when you take higher and higher order derivatives of $n^b$ and $a^n$: the polynomial vanishes by an order each time and the exponential is only multiplied by a constant factors ($ln a$) each time.
    $endgroup$
    – jnm2
    Aug 4 '11 at 2:36








5




5




$begingroup$
Have you tried expressing both in terms of the exponential?
$endgroup$
– gary
Aug 4 '11 at 1:41




$begingroup$
Have you tried expressing both in terms of the exponential?
$endgroup$
– gary
Aug 4 '11 at 1:41












$begingroup$
You could write the exponential as an infinite polynomial power series. You could also see what happens when you take higher and higher order derivatives of $n^b$ and $a^n$: the polynomial vanishes by an order each time and the exponential is only multiplied by a constant factors ($ln a$) each time.
$endgroup$
– jnm2
Aug 4 '11 at 2:36




$begingroup$
You could write the exponential as an infinite polynomial power series. You could also see what happens when you take higher and higher order derivatives of $n^b$ and $a^n$: the polynomial vanishes by an order each time and the exponential is only multiplied by a constant factors ($ln a$) each time.
$endgroup$
– jnm2
Aug 4 '11 at 2:36










13 Answers
13






active

oldest

votes


















27












$begingroup$

We could prove this by induction on integers $k$:



$$
lim_{n to infty} frac{n^k}{a^n} = 0.
$$



The case $k = 0$ is straightforward. I will leave the induction step to you. To see how this implies the statement for all real $b$, just note that every real number is less than some integer. In particular, $b leq lceil b rceil$. Thus,



$$
0 leq lim_{n to infty} frac{n^b}{a^n} leq lim_{n to infty} frac{n^{lceil b rceil}}{a^n} = 0.
$$



The first inequality follows since all the terms are positive. The last equality follows from the induction we established previously.






share|cite|improve this answer









$endgroup$













  • $begingroup$
    Could this really work? I mean $lim_{nrightarrowinfty}n = infty$. How can I do the induction?
    $endgroup$
    – ablmf
    Aug 4 '11 at 2:55












  • $begingroup$
    What you said is true, but I don't see how it enters the picture. As far as the induction process: when $k = 0$, you need to show $lim_{n to infty} frac{1}{a^n} =0$. As far as the induction step, assume that it holds for $k$, then show that it also holds for $k+1$ (Apply L'Hopital). This is pretty much the entire solution, but again, I will let you fill in the details.
    $endgroup$
    – JavaMan
    Aug 4 '11 at 3:27






  • 2




    $begingroup$
    Oh, I didn't know L'Hopital before. So the induction is $lim_{nrightarrowinfty}frac{n^(k+1)}{a^n} = lim_{nrightarrowinfty}frac{(k + 1) n^k}{ln(a)a^n} = 0$. Is that right?
    $endgroup$
    – ablmf
    Aug 4 '11 at 3:41












  • $begingroup$
    Yes that's right. Since $k+1$ and $ln(a)$ are constants, we can pull them out of the limit to get $frac{k+1}{ln(a)}lim_{n to infty} frac{n^{k}}{a^n} = frac{k+1}{ln(a)} cdot 0 = 0$.
    $endgroup$
    – JavaMan
    Aug 4 '11 at 3:45










  • $begingroup$
    Also, I should point out that when using TeX, if you enclose your exponent with braces like "a^{2x}", it will exponentiation the entire term and it will look like $a^{2x}$. Without braces, you get $a^2x$.
    $endgroup$
    – JavaMan
    Aug 4 '11 at 3:48





















39












$begingroup$

We can confine attention to $b ge 1$. This is because, if $0<b<1$, then $n^b le n$. If we can prove that $n/a^n$ approaches $0$, it will follow that $n^b/a^n$ approaches $0$ for any positive $ble 1$. So from now on we take $b ge 1$.



Now look at $n^b/a^n$, and take the $b$-th root. We get
$$frac{n}{(a^{1/b})^n}$$
or equivalently
$$frac{n}{c^n}$$
where $c=a^{1/b}$.



Note that $c>1$. If we can prove that $n/c^n$ approaches $0$ as $ntoinfty$, we will be finished.



For our original sequence consists of the $b$-th powers of the new sequence $(n/c^n)$. If we can show that $n/c^n$ has limit $0$, then after a while, $n/c^n le 1$, and so, after a while, the old sequence is, term by term, $le$ the new sequence. (Recall that $bge 1$.)



Progress, we need only look at $n/c^n$.



How do we continue? Any of the ways suggested by the other posts. Or else, let
$c=1+d$. Note that $d$ is positive. Note also that from the Binomial Theorem, if $n ge 2$ we have
$$c^n=(1+d)^n ge 1+dn+d^2n(n-1)/2gt d^2(n)(n-1)/2.$$



It follows that
$$frac{n}{c^n}< frac{n}{d^2(n)(n-1)/2}=frac{2}{d^2(n-1)}.$$
and it is clear that $dfrac{2}{d^2(n-1)} to 0$ as $ntoinfty$.






share|cite|improve this answer











$endgroup$









  • 8




    $begingroup$
    +1: I really, honestly and whole-heartedly think that this is the way to do this problem. From scratch. No l'Hospital, no properties of logarithms. Nothing shaky :-). Just the binomial theorem (or Bernoulli's inequality) to do the case $b=1$.
    $endgroup$
    – Jyrki Lahtonen
    Aug 4 '11 at 8:03










  • $begingroup$
    This is an elegant proof that could easily be understood. I will one day I could solve these problems as well as you do.
    $endgroup$
    – ablmf
    Aug 4 '11 at 14:04










  • $begingroup$
    Excellent answer.
    $endgroup$
    – YoTengoUnLCD
    Jun 9 '16 at 23:56



















18












$begingroup$

First, notice that $(n+1)^b/n^b=1$ plus terms in negative powers of $n$, so it goes to 1 as $ntoinfty$ (with $b$ fixed). Now going from $n^b/a^n$ to $(n+1)^b/a^{n+1}$, you multiply the numerator by something that's going to 1 but the denominator by something that isn't (namely, by $agt1$).






share|cite|improve this answer









$endgroup$









  • 3




    $begingroup$
    This is the simplest solution I can think of.
    $endgroup$
    – Qiaochu Yuan
    Aug 4 '11 at 2:59






  • 1




    $begingroup$
    This is a special case of multiplicative telescopy. See my post here for much further discussion of this example. Similar ideas work more generally for hyperrational functions.
    $endgroup$
    – Bill Dubuque
    Aug 4 '11 at 3:02












  • $begingroup$
    That means $lim_{nrightarrowinfty}frac{n^b}{a^n}=lim_{nrightarrowinfty}frac{(n+1)^b}{a^(n+1)}$, so it must be zero. Is my understanding right?
    $endgroup$
    – ablmf
    Aug 4 '11 at 3:18










  • $begingroup$
    Yes, that's one good way to understand it.
    $endgroup$
    – Gerry Myerson
    Aug 4 '11 at 3:50










  • $begingroup$
    @GerryMyerson I’m just reading this right now, but that’s a bad way to understand it: It isn’t clear yet that the limit exists at all and for any converging sequence $lim a_n = lim a_{n+1}$ holds, so one cannot deduce anything about the limit from this or am I missing something? Maybe ablmf meant $lim_{n → ∞} {n^b}/{a^n} = lim_{n → ∞} {n^b}/{a^{n+1}}$? But here again you first need to know that the limit exists.
    $endgroup$
    – k.stm
    Jun 2 '13 at 17:55





















9












$begingroup$

You can expand as$$a^n = e^{nlog a} = 1+nlog a +cdots +frac{(nlog a)^b}{b!}+cdots$$



$$frac{a^n}{n^b} =frac{1}{n^b}+frac{1}{n^{b-1}}log a+cdots+frac{(log a)^{b}}{b!}+nfrac{(log a)^{b+1}}{(b+1)!}+cdots $$
So, $$frac{a^n}{n^b}geq nfrac{(log a)^{b+1}}{(b+1)!}$$. This becomes arbitrarily large with large $n$



Therefore, $$lim_{nrightarrowinfty}frac{a^n}{n^b} = infty$$ Or $$lim_{nrightarrowinfty}frac{n^b}{a^n} = 0$$






share|cite|improve this answer











$endgroup$





















    3












    $begingroup$

    Regard the exponential function as the unique solution to the initial value problem $f(0) = 1$ with $f' = f$. The equivalent integral formulation is



    $f(t) = 1 + int_0^t f(s) ds$



    First prove that $f$ is an increasing function. For $t geq 0$, you can use a method of continuity argument by considering the set ${ s ~:~ f mbox{ increases on } [0, s] }$. The supremum of this set cannot be finite. For $s < 0$ you can also consider the ODE satisfied by $f^2$, but you're not really concerned with that. Now that we know $f$ increase, we also know $f$ grows at least linearly:



    $f(t) geq 1 + int_0^t 1 ds = (1 + t)$



    It also grows at least quadratically..



    $f(t) geq f(frac{t}{2}) + int_{t/2}^t f(s) ds geq (1 + t/2) f(t/2)$



    And repeating yields



    $f(t) geq (1 + t/2)^2$



    In general, $f(t) geq (1 + t/n)^n$ by the same method. It's even true for $n$ not an integer. In this case, the function $(1 + t/n)^n$ is defined in the first place to be $e^{n log( 1 + t/n) }$. Then, since log x is defined by the integral $int_1^x frac{1}{t} dt = x int_0^1 (1 + sx)^{-1} ds $, we can rewrite



    $(1 + t/n)^n = e^{n log (1 + t/n) } = mbox{exp}(t int_0^1 (1 + frac{s t}{n})^{-1} ds ) leq e^t$



    EDIT: This answer may not be so helpful to you. I didn't read the end of your question, so you may not know how to work with most of the things I've been writing about. The main reason why the exponential grows faster than a polynomial is because if $f$ is exponential, then $f(n+1)$ is at least a constant times $f(n)$, whereas when $f$ is a polynomial, $f(n+1)$ is roughly the same size as $f(n)$ when $n$ is large. After all, you can hardly tell the difference between the volume of a huge cube, and a huge cube 1 unit longer in each direction.






    share|cite|improve this answer









    $endgroup$













    • $begingroup$
      Oops, seems that I won't be able to understand this before I finish my calculus course.
      $endgroup$
      – ablmf
      Aug 4 '11 at 2:59



















    3












    $begingroup$

    Try expressing both in terms of the exponential $e^x$: $n^b$=$e^{bln(n)}$; $a^n=e^{nln(a)}$, so that the quotient becomes:



    $frac{n^b}{a^n}$= $frac{e^{bln(n)}}{e^{nln(a)}}=e^{bln(n)-nln(a)}=e^{bln(n)-kn}=frac{e^{nlna}}{e^{kn}}$ , where k is a real constant. Can you tell which of ln(n) or n grows faster?






    share|cite|improve this answer











    $endgroup$













    • $begingroup$
      So the question becomes to prove $lim_{nrightarrowinfty}(bln(n)-kn) = -infty$ right? BTW, you made some typos.
      $endgroup$
      – ablmf
      Aug 4 '11 at 2:58












    • $begingroup$
      Yes; I think too, if you use that ln(n) grows slower than n. I will check for typos.
      $endgroup$
      – gary
      Aug 4 '11 at 3:46





















    3












    $begingroup$

    This is inspired by André Nicolas's answer, but instead of taking $b^{th}$ roots, I'll take $n^{th}$ roots.



    The limit of the sequence of $n^{th}$ roots of the terms in the original sequence is
    $$lim_{ntoinfty}frac{(n^b)^{1/n}}{a}=lim_{ntoinfty}frac{left(n^{1/n}right)^b}{a}=frac{left(limlimits_{ntoinfty} n^{1/n}right)^b}{a}=frac{1^b}{a}=frac{1}{a}.$$



    (I used here the fact that $limlimits_{ntoinfty}n^{1/n}=1$, which was the subject of another question on this site, and which can also be proved in many ways.)



    If $c$ is any number such that $frac{1}{a}<c<1$, then the previous limit implies that there is an $N>0$ such that for all $n>N$, $$left(frac{n^b}{a^n}right)^{1/n}lt c.$$ Then for all $n>N$, $$frac{n^b}{a^n}lt c^n.$$ Since $c^nto 0$, this shows that the original series goes to zero.



    Alternatively, once you know that the sequence of $n^{th}$ roots converges to a number less than $1$, you can apply the root test to conclude that $sumlimits_{n=1}^inftyfrac{n^b}{a^n}$ converges, which implies that the terms go to $0$. (The proof of the root test actually just uses the inequality derived above and the convergence of the geometric series with common ratio $c$.)






    share|cite|improve this answer











    $endgroup$





















      3












      $begingroup$

      Since it is almost always better to expand around zero, let $a = 1+c$, where $c > 0$. So we want to show $lim frac{n^b}{(1+c)^n} = 0$.



      The ratio of consecutive terms is $frac{(1+1/n)^b}{1+c}$, so if we can show that $(1+1/n)^b < 1+c/2$ for large enough n, we are done. But this means that
      $n > 1/((1+c/2)^{1/b}-1)$.



      Restating, letting $N = frac{1}{(1+c/2)^{1/b}-1}$ and $r = frac{1+c/2}{1+c}$, if $n > N$ then $frac{(1+1/n)^b}{1+c} < frac{1+c/2}{1+c}$, which shows that
      $$ frac{n^b}{(1+c)^n} < frac{N^b}{(1+c)^N}r^{n-N}$$
      which goes to $0$ as $n rightarrow infty$ since $N$ is fixed and $0 < r < 1$.



      Note: An elementary proof that $r^n rightarrow 0$ is in "What is Mathematics?" by Courant and Robbins.



      Let $r = 1/(1+s)$, where $s > 0$. By Bernoulli's inequality,
      $$r^n = frac{1}{(1+s)^n} < frac{1}{1+s n} = frac{1}{1+ n (1/r-1)}.$$
      They similarly show that, if $a > 1$, then $a^n rightarrow infty$ like this:
      Let $ a = 1+b$, where $b > 0$. Then
      $$a^n = (1+b)^n > 1+n b = 1+n(a-1).$$
      The keys in both these are expanding around zero and using Bernoulli's inequality.
      Archimede's axiom (for any positive reals $x$ and $y$, there is an integer $n$ such that $x < n y$) does the rest.






      share|cite|improve this answer









      $endgroup$





















        1












        $begingroup$

        Tom Apostol, in volume I of his 2-volume text on calculus, gives what I consider to be the canonical proof of this assertion. He calls it the theorem of “the slow growth of the logarithm”. By making the appropriate substitution (namely, exp(x) for x), you can get what we can call the theorem on “the fast growth of the exponential”.



        I think you will have no difficulty agreeing that as far as comparing a polynomial to the exponential goes, the only part of the polynomial that we have to consider is the term involving the highest power, and that we can even disregard its coefficient. (This agreement is currently shown in your question, but I believe that is the result of someone else’s edit, and so I am answering what I believe your original question was.) That is what that theorem in Apostol does: it shows that if a > 0 and b > 0, then the limit as x goes to infinity of ((log(x))^a)/(x^b) goes to 0.



        You might be interested is knowing a significant application of this fact, namely, in the proof of the recursion formula for what is known as the Gamma function (which is the canonical continuous extension of the factorial function but, for historical reasons, shifted one unit). That is, in proving that Gamma(x + 1) = xGamma(x), one typically applies the technique of integration-by-parts, and one piece of the resulting expression is a power divided by an exponential, which goes to 0 as one takes the limit as the upper limit of the integral goes to infinity.



        Another application of this fact is the shape of the graph of y = x*exp(x), namely, that as x goes to negative infinity, y goes to 0.






        share|cite|improve this answer











        $endgroup$





















          1












          $begingroup$

          As several of the previous answers have noted it is enough to show that $lim_{xto infty} frac{x}{a^x}=0$. To prove this, in turn it is enough to prove that $lim_{x to infty} frac{a^x}{x} = infty$.



          Lets compute the derivative and second derivative of $f(x) = frac{a^x}{x}$. We find that
          $f'(x) = frac{a^x(x ln(a)-1)}{x^2}$ and $f''(x) = frac{a^x(x^2 ln^2(a)-2ln(a) x+2)}{x^3}$.



          We see that for large enough values of $x$, both $f'(x)$ and $f''(x)$ are positive. Thus, $f$ not only grows, it grows at an increasing rate. The only possible conclusion is that $lim_{xto infty} frac{a^x}{x} = infty$.



          With a little work this can be made into a completely rigorous proof.






          share|cite|improve this answer









          $endgroup$





















            1












            $begingroup$

            Another method is the following (this method was originally? used by W. Rudin in his PMA):



            For $ngeqslant2(|lceil brceil|+1)$ (the absolute value of the ceiling of $b$) we have, using the binomial theorem:$$begin{aligned}a^n&=(1+(a-1))^n\\&>binom{n}{|lceil brceil|+1}(a-1)^{|lceil brceil|+1}\\&=dfrac{n(n-1)cdots(n-|lceil brceil|)}{(|lceil brceil|+1)!}(a-1)^{|lceil brceil|+1}\\&>dfrac{n^{|lceil brceil|+1}}{2^{|lceil brceil|+1}(|lceil brceil|+1)!}(a-1)^{|lceil brceil|+1}end{aligned}$$ so $$left(dfrac{2^{|lceil brceil|+1}(|lceil brceil|+1)!}{(a-1)^{|lceil brceil|+1}}right)cdotdfrac{1}{n}>dfrac{n^{|lceil brceil|}}{a^n}geqslantdfrac{n^b}{a^n}$$ and hence $$lim_{ntoinfty}dfrac{n^b}{a^n}=0.$$






            share|cite|improve this answer









            $endgroup$





















              1












              $begingroup$

              The case $ ble 0$ is blatantly obvious since $n^b le 1$ for $ble 0$ and hence since $a>1$ we have, $$frac{n^b}{a^n} le frac{1}{a^n} =left(frac{1}{a}right)^nto 0$$



              we assume $ b>0$ then,



              $$frac{n^b}{a^n}=left(frac{b}{ln a} right)^bleft(nfrac{ln a}{b} e^{-nfrac{ln a}{b}}right)^b color{red}{overset{u= nfrac{ln a}{b}}{:=}left(frac{b}{ln a} right)^bleft(u e^{-u}right)^b} ~~~~ b>0,~~$$



              Hence, $$lim_{ntoinfty}frac{n^b}{a^n}= lim_{utoinfty}left(frac{b}{ln a} right)^bleft(u e^{-u}right)^b=0$$









              share|cite|improve this answer











              $endgroup$





















                1












                $begingroup$

                You could also see the following:
                $$lim_{nrightarrowinfty}frac{n^b}{a^n} = lim_{nrightarrowinfty}e^{log frac{n^b}{a^n}}$$



                Now $lim_{nrightarrowinfty}log (frac{n^b}{a^n})=lim_{nrightarrowinfty}(log (n^b) - log (a^n))=lim_{nrightarrowinfty} (blog(n)-nlog(a))=-infty$ assuming $a>1$.



                So $lim_{nrightarrowinfty}e^{log frac{n^b}{a^n}}=0=lim_{nrightarrowinfty}frac{n^b}{a^n}$



                Just to clarify the comment, I am adding the proof that $$lim_{nrightarrowinfty} (blog (n)-nlog (a))=-infty$$



                First by applying L'Hôpital's rule we get



                $$lim_{nrightarrowinfty} frac{n}{log(n)}=lim_{nrightarrowinfty} frac{1}{frac{1}{n}} to infty$$
                We have then
                $$lim_{nrightarrowinfty} (blog (n)-nlog (a))=lim_{nrightarrowinfty} log(n)(b-log(a)frac{n}{log(n)})=-infty$$






                share|cite|improve this answer











                $endgroup$













                • $begingroup$
                  Then you need an upper bound of $log(n)$.
                  $endgroup$
                  – ablmf
                  Apr 15 '18 at 9:18










                • $begingroup$
                  I assumed obviously that is a known fact that $log (n) to infty$ as $n to infty$ and that you know how to prove $lim_{nrightarrowinfty} (blog n-nlog a)=-infty$ assuming $a>1$
                  $endgroup$
                  – Marco Bellocchi
                  Apr 15 '18 at 10:03














                Your Answer








                StackExchange.ready(function() {
                var channelOptions = {
                tags: "".split(" "),
                id: "69"
                };
                initTagRenderer("".split(" "), "".split(" "), channelOptions);

                StackExchange.using("externalEditor", function() {
                // Have to fire editor after snippets, if snippets enabled
                if (StackExchange.settings.snippets.snippetsEnabled) {
                StackExchange.using("snippets", function() {
                createEditor();
                });
                }
                else {
                createEditor();
                }
                });

                function createEditor() {
                StackExchange.prepareEditor({
                heartbeatType: 'answer',
                autoActivateHeartbeat: false,
                convertImagesToLinks: true,
                noModals: true,
                showLowRepImageUploadWarning: true,
                reputationToPostImages: 10,
                bindNavPrevention: true,
                postfix: "",
                imageUploader: {
                brandingHtml: "Powered by u003ca class="icon-imgur-white" href="https://imgur.com/"u003eu003c/au003e",
                contentPolicyHtml: "User contributions licensed under u003ca href="https://creativecommons.org/licenses/by-sa/3.0/"u003ecc by-sa 3.0 with attribution requiredu003c/au003e u003ca href="https://stackoverflow.com/legal/content-policy"u003e(content policy)u003c/au003e",
                allowUrls: true
                },
                noCode: true, onDemand: true,
                discardSelector: ".discard-answer"
                ,immediatelyShowMarkdownHelp:true
                });


                }
                });














                draft saved

                draft discarded


















                StackExchange.ready(
                function () {
                StackExchange.openid.initPostLogin('.new-post-login', 'https%3a%2f%2fmath.stackexchange.com%2fquestions%2f55468%2fhow-to-prove-that-exponential-grows-faster-than-polynomial%23new-answer', 'question_page');
                }
                );

                Post as a guest















                Required, but never shown

























                13 Answers
                13






                active

                oldest

                votes








                13 Answers
                13






                active

                oldest

                votes









                active

                oldest

                votes






                active

                oldest

                votes









                27












                $begingroup$

                We could prove this by induction on integers $k$:



                $$
                lim_{n to infty} frac{n^k}{a^n} = 0.
                $$



                The case $k = 0$ is straightforward. I will leave the induction step to you. To see how this implies the statement for all real $b$, just note that every real number is less than some integer. In particular, $b leq lceil b rceil$. Thus,



                $$
                0 leq lim_{n to infty} frac{n^b}{a^n} leq lim_{n to infty} frac{n^{lceil b rceil}}{a^n} = 0.
                $$



                The first inequality follows since all the terms are positive. The last equality follows from the induction we established previously.






                share|cite|improve this answer









                $endgroup$













                • $begingroup$
                  Could this really work? I mean $lim_{nrightarrowinfty}n = infty$. How can I do the induction?
                  $endgroup$
                  – ablmf
                  Aug 4 '11 at 2:55












                • $begingroup$
                  What you said is true, but I don't see how it enters the picture. As far as the induction process: when $k = 0$, you need to show $lim_{n to infty} frac{1}{a^n} =0$. As far as the induction step, assume that it holds for $k$, then show that it also holds for $k+1$ (Apply L'Hopital). This is pretty much the entire solution, but again, I will let you fill in the details.
                  $endgroup$
                  – JavaMan
                  Aug 4 '11 at 3:27






                • 2




                  $begingroup$
                  Oh, I didn't know L'Hopital before. So the induction is $lim_{nrightarrowinfty}frac{n^(k+1)}{a^n} = lim_{nrightarrowinfty}frac{(k + 1) n^k}{ln(a)a^n} = 0$. Is that right?
                  $endgroup$
                  – ablmf
                  Aug 4 '11 at 3:41












                • $begingroup$
                  Yes that's right. Since $k+1$ and $ln(a)$ are constants, we can pull them out of the limit to get $frac{k+1}{ln(a)}lim_{n to infty} frac{n^{k}}{a^n} = frac{k+1}{ln(a)} cdot 0 = 0$.
                  $endgroup$
                  – JavaMan
                  Aug 4 '11 at 3:45










                • $begingroup$
                  Also, I should point out that when using TeX, if you enclose your exponent with braces like "a^{2x}", it will exponentiation the entire term and it will look like $a^{2x}$. Without braces, you get $a^2x$.
                  $endgroup$
                  – JavaMan
                  Aug 4 '11 at 3:48


















                27












                $begingroup$

                We could prove this by induction on integers $k$:



                $$
                lim_{n to infty} frac{n^k}{a^n} = 0.
                $$



                The case $k = 0$ is straightforward. I will leave the induction step to you. To see how this implies the statement for all real $b$, just note that every real number is less than some integer. In particular, $b leq lceil b rceil$. Thus,



                $$
                0 leq lim_{n to infty} frac{n^b}{a^n} leq lim_{n to infty} frac{n^{lceil b rceil}}{a^n} = 0.
                $$



                The first inequality follows since all the terms are positive. The last equality follows from the induction we established previously.






                share|cite|improve this answer









                $endgroup$













                • $begingroup$
                  Could this really work? I mean $lim_{nrightarrowinfty}n = infty$. How can I do the induction?
                  $endgroup$
                  – ablmf
                  Aug 4 '11 at 2:55












                • $begingroup$
                  What you said is true, but I don't see how it enters the picture. As far as the induction process: when $k = 0$, you need to show $lim_{n to infty} frac{1}{a^n} =0$. As far as the induction step, assume that it holds for $k$, then show that it also holds for $k+1$ (Apply L'Hopital). This is pretty much the entire solution, but again, I will let you fill in the details.
                  $endgroup$
                  – JavaMan
                  Aug 4 '11 at 3:27






                • 2




                  $begingroup$
                  Oh, I didn't know L'Hopital before. So the induction is $lim_{nrightarrowinfty}frac{n^(k+1)}{a^n} = lim_{nrightarrowinfty}frac{(k + 1) n^k}{ln(a)a^n} = 0$. Is that right?
                  $endgroup$
                  – ablmf
                  Aug 4 '11 at 3:41












                • $begingroup$
                  Yes that's right. Since $k+1$ and $ln(a)$ are constants, we can pull them out of the limit to get $frac{k+1}{ln(a)}lim_{n to infty} frac{n^{k}}{a^n} = frac{k+1}{ln(a)} cdot 0 = 0$.
                  $endgroup$
                  – JavaMan
                  Aug 4 '11 at 3:45










                • $begingroup$
                  Also, I should point out that when using TeX, if you enclose your exponent with braces like "a^{2x}", it will exponentiation the entire term and it will look like $a^{2x}$. Without braces, you get $a^2x$.
                  $endgroup$
                  – JavaMan
                  Aug 4 '11 at 3:48
















                27












                27








                27





                $begingroup$

                We could prove this by induction on integers $k$:



                $$
                lim_{n to infty} frac{n^k}{a^n} = 0.
                $$



                The case $k = 0$ is straightforward. I will leave the induction step to you. To see how this implies the statement for all real $b$, just note that every real number is less than some integer. In particular, $b leq lceil b rceil$. Thus,



                $$
                0 leq lim_{n to infty} frac{n^b}{a^n} leq lim_{n to infty} frac{n^{lceil b rceil}}{a^n} = 0.
                $$



                The first inequality follows since all the terms are positive. The last equality follows from the induction we established previously.






                share|cite|improve this answer









                $endgroup$



                We could prove this by induction on integers $k$:



                $$
                lim_{n to infty} frac{n^k}{a^n} = 0.
                $$



                The case $k = 0$ is straightforward. I will leave the induction step to you. To see how this implies the statement for all real $b$, just note that every real number is less than some integer. In particular, $b leq lceil b rceil$. Thus,



                $$
                0 leq lim_{n to infty} frac{n^b}{a^n} leq lim_{n to infty} frac{n^{lceil b rceil}}{a^n} = 0.
                $$



                The first inequality follows since all the terms are positive. The last equality follows from the induction we established previously.







                share|cite|improve this answer












                share|cite|improve this answer



                share|cite|improve this answer










                answered Aug 4 '11 at 2:38









                JavaManJavaMan

                11.2k12855




                11.2k12855












                • $begingroup$
                  Could this really work? I mean $lim_{nrightarrowinfty}n = infty$. How can I do the induction?
                  $endgroup$
                  – ablmf
                  Aug 4 '11 at 2:55












                • $begingroup$
                  What you said is true, but I don't see how it enters the picture. As far as the induction process: when $k = 0$, you need to show $lim_{n to infty} frac{1}{a^n} =0$. As far as the induction step, assume that it holds for $k$, then show that it also holds for $k+1$ (Apply L'Hopital). This is pretty much the entire solution, but again, I will let you fill in the details.
                  $endgroup$
                  – JavaMan
                  Aug 4 '11 at 3:27






                • 2




                  $begingroup$
                  Oh, I didn't know L'Hopital before. So the induction is $lim_{nrightarrowinfty}frac{n^(k+1)}{a^n} = lim_{nrightarrowinfty}frac{(k + 1) n^k}{ln(a)a^n} = 0$. Is that right?
                  $endgroup$
                  – ablmf
                  Aug 4 '11 at 3:41












                • $begingroup$
                  Yes that's right. Since $k+1$ and $ln(a)$ are constants, we can pull them out of the limit to get $frac{k+1}{ln(a)}lim_{n to infty} frac{n^{k}}{a^n} = frac{k+1}{ln(a)} cdot 0 = 0$.
                  $endgroup$
                  – JavaMan
                  Aug 4 '11 at 3:45










                • $begingroup$
                  Also, I should point out that when using TeX, if you enclose your exponent with braces like "a^{2x}", it will exponentiation the entire term and it will look like $a^{2x}$. Without braces, you get $a^2x$.
                  $endgroup$
                  – JavaMan
                  Aug 4 '11 at 3:48




















                • $begingroup$
                  Could this really work? I mean $lim_{nrightarrowinfty}n = infty$. How can I do the induction?
                  $endgroup$
                  – ablmf
                  Aug 4 '11 at 2:55












                • $begingroup$
                  What you said is true, but I don't see how it enters the picture. As far as the induction process: when $k = 0$, you need to show $lim_{n to infty} frac{1}{a^n} =0$. As far as the induction step, assume that it holds for $k$, then show that it also holds for $k+1$ (Apply L'Hopital). This is pretty much the entire solution, but again, I will let you fill in the details.
                  $endgroup$
                  – JavaMan
                  Aug 4 '11 at 3:27






                • 2




                  $begingroup$
                  Oh, I didn't know L'Hopital before. So the induction is $lim_{nrightarrowinfty}frac{n^(k+1)}{a^n} = lim_{nrightarrowinfty}frac{(k + 1) n^k}{ln(a)a^n} = 0$. Is that right?
                  $endgroup$
                  – ablmf
                  Aug 4 '11 at 3:41












                • $begingroup$
                  Yes that's right. Since $k+1$ and $ln(a)$ are constants, we can pull them out of the limit to get $frac{k+1}{ln(a)}lim_{n to infty} frac{n^{k}}{a^n} = frac{k+1}{ln(a)} cdot 0 = 0$.
                  $endgroup$
                  – JavaMan
                  Aug 4 '11 at 3:45










                • $begingroup$
                  Also, I should point out that when using TeX, if you enclose your exponent with braces like "a^{2x}", it will exponentiation the entire term and it will look like $a^{2x}$. Without braces, you get $a^2x$.
                  $endgroup$
                  – JavaMan
                  Aug 4 '11 at 3:48


















                $begingroup$
                Could this really work? I mean $lim_{nrightarrowinfty}n = infty$. How can I do the induction?
                $endgroup$
                – ablmf
                Aug 4 '11 at 2:55






                $begingroup$
                Could this really work? I mean $lim_{nrightarrowinfty}n = infty$. How can I do the induction?
                $endgroup$
                – ablmf
                Aug 4 '11 at 2:55














                $begingroup$
                What you said is true, but I don't see how it enters the picture. As far as the induction process: when $k = 0$, you need to show $lim_{n to infty} frac{1}{a^n} =0$. As far as the induction step, assume that it holds for $k$, then show that it also holds for $k+1$ (Apply L'Hopital). This is pretty much the entire solution, but again, I will let you fill in the details.
                $endgroup$
                – JavaMan
                Aug 4 '11 at 3:27




                $begingroup$
                What you said is true, but I don't see how it enters the picture. As far as the induction process: when $k = 0$, you need to show $lim_{n to infty} frac{1}{a^n} =0$. As far as the induction step, assume that it holds for $k$, then show that it also holds for $k+1$ (Apply L'Hopital). This is pretty much the entire solution, but again, I will let you fill in the details.
                $endgroup$
                – JavaMan
                Aug 4 '11 at 3:27




                2




                2




                $begingroup$
                Oh, I didn't know L'Hopital before. So the induction is $lim_{nrightarrowinfty}frac{n^(k+1)}{a^n} = lim_{nrightarrowinfty}frac{(k + 1) n^k}{ln(a)a^n} = 0$. Is that right?
                $endgroup$
                – ablmf
                Aug 4 '11 at 3:41






                $begingroup$
                Oh, I didn't know L'Hopital before. So the induction is $lim_{nrightarrowinfty}frac{n^(k+1)}{a^n} = lim_{nrightarrowinfty}frac{(k + 1) n^k}{ln(a)a^n} = 0$. Is that right?
                $endgroup$
                – ablmf
                Aug 4 '11 at 3:41














                $begingroup$
                Yes that's right. Since $k+1$ and $ln(a)$ are constants, we can pull them out of the limit to get $frac{k+1}{ln(a)}lim_{n to infty} frac{n^{k}}{a^n} = frac{k+1}{ln(a)} cdot 0 = 0$.
                $endgroup$
                – JavaMan
                Aug 4 '11 at 3:45




                $begingroup$
                Yes that's right. Since $k+1$ and $ln(a)$ are constants, we can pull them out of the limit to get $frac{k+1}{ln(a)}lim_{n to infty} frac{n^{k}}{a^n} = frac{k+1}{ln(a)} cdot 0 = 0$.
                $endgroup$
                – JavaMan
                Aug 4 '11 at 3:45












                $begingroup$
                Also, I should point out that when using TeX, if you enclose your exponent with braces like "a^{2x}", it will exponentiation the entire term and it will look like $a^{2x}$. Without braces, you get $a^2x$.
                $endgroup$
                – JavaMan
                Aug 4 '11 at 3:48






                $begingroup$
                Also, I should point out that when using TeX, if you enclose your exponent with braces like "a^{2x}", it will exponentiation the entire term and it will look like $a^{2x}$. Without braces, you get $a^2x$.
                $endgroup$
                – JavaMan
                Aug 4 '11 at 3:48













                39












                $begingroup$

                We can confine attention to $b ge 1$. This is because, if $0<b<1$, then $n^b le n$. If we can prove that $n/a^n$ approaches $0$, it will follow that $n^b/a^n$ approaches $0$ for any positive $ble 1$. So from now on we take $b ge 1$.



                Now look at $n^b/a^n$, and take the $b$-th root. We get
                $$frac{n}{(a^{1/b})^n}$$
                or equivalently
                $$frac{n}{c^n}$$
                where $c=a^{1/b}$.



                Note that $c>1$. If we can prove that $n/c^n$ approaches $0$ as $ntoinfty$, we will be finished.



                For our original sequence consists of the $b$-th powers of the new sequence $(n/c^n)$. If we can show that $n/c^n$ has limit $0$, then after a while, $n/c^n le 1$, and so, after a while, the old sequence is, term by term, $le$ the new sequence. (Recall that $bge 1$.)



                Progress, we need only look at $n/c^n$.



                How do we continue? Any of the ways suggested by the other posts. Or else, let
                $c=1+d$. Note that $d$ is positive. Note also that from the Binomial Theorem, if $n ge 2$ we have
                $$c^n=(1+d)^n ge 1+dn+d^2n(n-1)/2gt d^2(n)(n-1)/2.$$



                It follows that
                $$frac{n}{c^n}< frac{n}{d^2(n)(n-1)/2}=frac{2}{d^2(n-1)}.$$
                and it is clear that $dfrac{2}{d^2(n-1)} to 0$ as $ntoinfty$.






                share|cite|improve this answer











                $endgroup$









                • 8




                  $begingroup$
                  +1: I really, honestly and whole-heartedly think that this is the way to do this problem. From scratch. No l'Hospital, no properties of logarithms. Nothing shaky :-). Just the binomial theorem (or Bernoulli's inequality) to do the case $b=1$.
                  $endgroup$
                  – Jyrki Lahtonen
                  Aug 4 '11 at 8:03










                • $begingroup$
                  This is an elegant proof that could easily be understood. I will one day I could solve these problems as well as you do.
                  $endgroup$
                  – ablmf
                  Aug 4 '11 at 14:04










                • $begingroup$
                  Excellent answer.
                  $endgroup$
                  – YoTengoUnLCD
                  Jun 9 '16 at 23:56
















                39












                $begingroup$

                We can confine attention to $b ge 1$. This is because, if $0<b<1$, then $n^b le n$. If we can prove that $n/a^n$ approaches $0$, it will follow that $n^b/a^n$ approaches $0$ for any positive $ble 1$. So from now on we take $b ge 1$.



                Now look at $n^b/a^n$, and take the $b$-th root. We get
                $$frac{n}{(a^{1/b})^n}$$
                or equivalently
                $$frac{n}{c^n}$$
                where $c=a^{1/b}$.



                Note that $c>1$. If we can prove that $n/c^n$ approaches $0$ as $ntoinfty$, we will be finished.



                For our original sequence consists of the $b$-th powers of the new sequence $(n/c^n)$. If we can show that $n/c^n$ has limit $0$, then after a while, $n/c^n le 1$, and so, after a while, the old sequence is, term by term, $le$ the new sequence. (Recall that $bge 1$.)



                Progress, we need only look at $n/c^n$.



                How do we continue? Any of the ways suggested by the other posts. Or else, let
                $c=1+d$. Note that $d$ is positive. Note also that from the Binomial Theorem, if $n ge 2$ we have
                $$c^n=(1+d)^n ge 1+dn+d^2n(n-1)/2gt d^2(n)(n-1)/2.$$



                It follows that
                $$frac{n}{c^n}< frac{n}{d^2(n)(n-1)/2}=frac{2}{d^2(n-1)}.$$
                and it is clear that $dfrac{2}{d^2(n-1)} to 0$ as $ntoinfty$.






                share|cite|improve this answer











                $endgroup$









                • 8




                  $begingroup$
                  +1: I really, honestly and whole-heartedly think that this is the way to do this problem. From scratch. No l'Hospital, no properties of logarithms. Nothing shaky :-). Just the binomial theorem (or Bernoulli's inequality) to do the case $b=1$.
                  $endgroup$
                  – Jyrki Lahtonen
                  Aug 4 '11 at 8:03










                • $begingroup$
                  This is an elegant proof that could easily be understood. I will one day I could solve these problems as well as you do.
                  $endgroup$
                  – ablmf
                  Aug 4 '11 at 14:04










                • $begingroup$
                  Excellent answer.
                  $endgroup$
                  – YoTengoUnLCD
                  Jun 9 '16 at 23:56














                39












                39








                39





                $begingroup$

                We can confine attention to $b ge 1$. This is because, if $0<b<1$, then $n^b le n$. If we can prove that $n/a^n$ approaches $0$, it will follow that $n^b/a^n$ approaches $0$ for any positive $ble 1$. So from now on we take $b ge 1$.



                Now look at $n^b/a^n$, and take the $b$-th root. We get
                $$frac{n}{(a^{1/b})^n}$$
                or equivalently
                $$frac{n}{c^n}$$
                where $c=a^{1/b}$.



                Note that $c>1$. If we can prove that $n/c^n$ approaches $0$ as $ntoinfty$, we will be finished.



                For our original sequence consists of the $b$-th powers of the new sequence $(n/c^n)$. If we can show that $n/c^n$ has limit $0$, then after a while, $n/c^n le 1$, and so, after a while, the old sequence is, term by term, $le$ the new sequence. (Recall that $bge 1$.)



                Progress, we need only look at $n/c^n$.



                How do we continue? Any of the ways suggested by the other posts. Or else, let
                $c=1+d$. Note that $d$ is positive. Note also that from the Binomial Theorem, if $n ge 2$ we have
                $$c^n=(1+d)^n ge 1+dn+d^2n(n-1)/2gt d^2(n)(n-1)/2.$$



                It follows that
                $$frac{n}{c^n}< frac{n}{d^2(n)(n-1)/2}=frac{2}{d^2(n-1)}.$$
                and it is clear that $dfrac{2}{d^2(n-1)} to 0$ as $ntoinfty$.






                share|cite|improve this answer











                $endgroup$



                We can confine attention to $b ge 1$. This is because, if $0<b<1$, then $n^b le n$. If we can prove that $n/a^n$ approaches $0$, it will follow that $n^b/a^n$ approaches $0$ for any positive $ble 1$. So from now on we take $b ge 1$.



                Now look at $n^b/a^n$, and take the $b$-th root. We get
                $$frac{n}{(a^{1/b})^n}$$
                or equivalently
                $$frac{n}{c^n}$$
                where $c=a^{1/b}$.



                Note that $c>1$. If we can prove that $n/c^n$ approaches $0$ as $ntoinfty$, we will be finished.



                For our original sequence consists of the $b$-th powers of the new sequence $(n/c^n)$. If we can show that $n/c^n$ has limit $0$, then after a while, $n/c^n le 1$, and so, after a while, the old sequence is, term by term, $le$ the new sequence. (Recall that $bge 1$.)



                Progress, we need only look at $n/c^n$.



                How do we continue? Any of the ways suggested by the other posts. Or else, let
                $c=1+d$. Note that $d$ is positive. Note also that from the Binomial Theorem, if $n ge 2$ we have
                $$c^n=(1+d)^n ge 1+dn+d^2n(n-1)/2gt d^2(n)(n-1)/2.$$



                It follows that
                $$frac{n}{c^n}< frac{n}{d^2(n)(n-1)/2}=frac{2}{d^2(n-1)}.$$
                and it is clear that $dfrac{2}{d^2(n-1)} to 0$ as $ntoinfty$.







                share|cite|improve this answer














                share|cite|improve this answer



                share|cite|improve this answer








                edited Aug 4 '11 at 5:27

























                answered Aug 4 '11 at 3:23









                André NicolasAndré Nicolas

                455k36432822




                455k36432822








                • 8




                  $begingroup$
                  +1: I really, honestly and whole-heartedly think that this is the way to do this problem. From scratch. No l'Hospital, no properties of logarithms. Nothing shaky :-). Just the binomial theorem (or Bernoulli's inequality) to do the case $b=1$.
                  $endgroup$
                  – Jyrki Lahtonen
                  Aug 4 '11 at 8:03










                • $begingroup$
                  This is an elegant proof that could easily be understood. I will one day I could solve these problems as well as you do.
                  $endgroup$
                  – ablmf
                  Aug 4 '11 at 14:04










                • $begingroup$
                  Excellent answer.
                  $endgroup$
                  – YoTengoUnLCD
                  Jun 9 '16 at 23:56














                • 8




                  $begingroup$
                  +1: I really, honestly and whole-heartedly think that this is the way to do this problem. From scratch. No l'Hospital, no properties of logarithms. Nothing shaky :-). Just the binomial theorem (or Bernoulli's inequality) to do the case $b=1$.
                  $endgroup$
                  – Jyrki Lahtonen
                  Aug 4 '11 at 8:03










                • $begingroup$
                  This is an elegant proof that could easily be understood. I will one day I could solve these problems as well as you do.
                  $endgroup$
                  – ablmf
                  Aug 4 '11 at 14:04










                • $begingroup$
                  Excellent answer.
                  $endgroup$
                  – YoTengoUnLCD
                  Jun 9 '16 at 23:56








                8




                8




                $begingroup$
                +1: I really, honestly and whole-heartedly think that this is the way to do this problem. From scratch. No l'Hospital, no properties of logarithms. Nothing shaky :-). Just the binomial theorem (or Bernoulli's inequality) to do the case $b=1$.
                $endgroup$
                – Jyrki Lahtonen
                Aug 4 '11 at 8:03




                $begingroup$
                +1: I really, honestly and whole-heartedly think that this is the way to do this problem. From scratch. No l'Hospital, no properties of logarithms. Nothing shaky :-). Just the binomial theorem (or Bernoulli's inequality) to do the case $b=1$.
                $endgroup$
                – Jyrki Lahtonen
                Aug 4 '11 at 8:03












                $begingroup$
                This is an elegant proof that could easily be understood. I will one day I could solve these problems as well as you do.
                $endgroup$
                – ablmf
                Aug 4 '11 at 14:04




                $begingroup$
                This is an elegant proof that could easily be understood. I will one day I could solve these problems as well as you do.
                $endgroup$
                – ablmf
                Aug 4 '11 at 14:04












                $begingroup$
                Excellent answer.
                $endgroup$
                – YoTengoUnLCD
                Jun 9 '16 at 23:56




                $begingroup$
                Excellent answer.
                $endgroup$
                – YoTengoUnLCD
                Jun 9 '16 at 23:56











                18












                $begingroup$

                First, notice that $(n+1)^b/n^b=1$ plus terms in negative powers of $n$, so it goes to 1 as $ntoinfty$ (with $b$ fixed). Now going from $n^b/a^n$ to $(n+1)^b/a^{n+1}$, you multiply the numerator by something that's going to 1 but the denominator by something that isn't (namely, by $agt1$).






                share|cite|improve this answer









                $endgroup$









                • 3




                  $begingroup$
                  This is the simplest solution I can think of.
                  $endgroup$
                  – Qiaochu Yuan
                  Aug 4 '11 at 2:59






                • 1




                  $begingroup$
                  This is a special case of multiplicative telescopy. See my post here for much further discussion of this example. Similar ideas work more generally for hyperrational functions.
                  $endgroup$
                  – Bill Dubuque
                  Aug 4 '11 at 3:02












                • $begingroup$
                  That means $lim_{nrightarrowinfty}frac{n^b}{a^n}=lim_{nrightarrowinfty}frac{(n+1)^b}{a^(n+1)}$, so it must be zero. Is my understanding right?
                  $endgroup$
                  – ablmf
                  Aug 4 '11 at 3:18










                • $begingroup$
                  Yes, that's one good way to understand it.
                  $endgroup$
                  – Gerry Myerson
                  Aug 4 '11 at 3:50










                • $begingroup$
                  @GerryMyerson I’m just reading this right now, but that’s a bad way to understand it: It isn’t clear yet that the limit exists at all and for any converging sequence $lim a_n = lim a_{n+1}$ holds, so one cannot deduce anything about the limit from this or am I missing something? Maybe ablmf meant $lim_{n → ∞} {n^b}/{a^n} = lim_{n → ∞} {n^b}/{a^{n+1}}$? But here again you first need to know that the limit exists.
                  $endgroup$
                  – k.stm
                  Jun 2 '13 at 17:55


















                18












                $begingroup$

                First, notice that $(n+1)^b/n^b=1$ plus terms in negative powers of $n$, so it goes to 1 as $ntoinfty$ (with $b$ fixed). Now going from $n^b/a^n$ to $(n+1)^b/a^{n+1}$, you multiply the numerator by something that's going to 1 but the denominator by something that isn't (namely, by $agt1$).






                share|cite|improve this answer









                $endgroup$









                • 3




                  $begingroup$
                  This is the simplest solution I can think of.
                  $endgroup$
                  – Qiaochu Yuan
                  Aug 4 '11 at 2:59






                • 1




                  $begingroup$
                  This is a special case of multiplicative telescopy. See my post here for much further discussion of this example. Similar ideas work more generally for hyperrational functions.
                  $endgroup$
                  – Bill Dubuque
                  Aug 4 '11 at 3:02












                • $begingroup$
                  That means $lim_{nrightarrowinfty}frac{n^b}{a^n}=lim_{nrightarrowinfty}frac{(n+1)^b}{a^(n+1)}$, so it must be zero. Is my understanding right?
                  $endgroup$
                  – ablmf
                  Aug 4 '11 at 3:18










                • $begingroup$
                  Yes, that's one good way to understand it.
                  $endgroup$
                  – Gerry Myerson
                  Aug 4 '11 at 3:50










                • $begingroup$
                  @GerryMyerson I’m just reading this right now, but that’s a bad way to understand it: It isn’t clear yet that the limit exists at all and for any converging sequence $lim a_n = lim a_{n+1}$ holds, so one cannot deduce anything about the limit from this or am I missing something? Maybe ablmf meant $lim_{n → ∞} {n^b}/{a^n} = lim_{n → ∞} {n^b}/{a^{n+1}}$? But here again you first need to know that the limit exists.
                  $endgroup$
                  – k.stm
                  Jun 2 '13 at 17:55
















                18












                18








                18





                $begingroup$

                First, notice that $(n+1)^b/n^b=1$ plus terms in negative powers of $n$, so it goes to 1 as $ntoinfty$ (with $b$ fixed). Now going from $n^b/a^n$ to $(n+1)^b/a^{n+1}$, you multiply the numerator by something that's going to 1 but the denominator by something that isn't (namely, by $agt1$).






                share|cite|improve this answer









                $endgroup$



                First, notice that $(n+1)^b/n^b=1$ plus terms in negative powers of $n$, so it goes to 1 as $ntoinfty$ (with $b$ fixed). Now going from $n^b/a^n$ to $(n+1)^b/a^{n+1}$, you multiply the numerator by something that's going to 1 but the denominator by something that isn't (namely, by $agt1$).







                share|cite|improve this answer












                share|cite|improve this answer



                share|cite|improve this answer










                answered Aug 4 '11 at 1:59









                Gerry MyersonGerry Myerson

                148k8152306




                148k8152306








                • 3




                  $begingroup$
                  This is the simplest solution I can think of.
                  $endgroup$
                  – Qiaochu Yuan
                  Aug 4 '11 at 2:59






                • 1




                  $begingroup$
                  This is a special case of multiplicative telescopy. See my post here for much further discussion of this example. Similar ideas work more generally for hyperrational functions.
                  $endgroup$
                  – Bill Dubuque
                  Aug 4 '11 at 3:02












                • $begingroup$
                  That means $lim_{nrightarrowinfty}frac{n^b}{a^n}=lim_{nrightarrowinfty}frac{(n+1)^b}{a^(n+1)}$, so it must be zero. Is my understanding right?
                  $endgroup$
                  – ablmf
                  Aug 4 '11 at 3:18










                • $begingroup$
                  Yes, that's one good way to understand it.
                  $endgroup$
                  – Gerry Myerson
                  Aug 4 '11 at 3:50










                • $begingroup$
                  @GerryMyerson I’m just reading this right now, but that’s a bad way to understand it: It isn’t clear yet that the limit exists at all and for any converging sequence $lim a_n = lim a_{n+1}$ holds, so one cannot deduce anything about the limit from this or am I missing something? Maybe ablmf meant $lim_{n → ∞} {n^b}/{a^n} = lim_{n → ∞} {n^b}/{a^{n+1}}$? But here again you first need to know that the limit exists.
                  $endgroup$
                  – k.stm
                  Jun 2 '13 at 17:55
















                • 3




                  $begingroup$
                  This is the simplest solution I can think of.
                  $endgroup$
                  – Qiaochu Yuan
                  Aug 4 '11 at 2:59






                • 1




                  $begingroup$
                  This is a special case of multiplicative telescopy. See my post here for much further discussion of this example. Similar ideas work more generally for hyperrational functions.
                  $endgroup$
                  – Bill Dubuque
                  Aug 4 '11 at 3:02












                • $begingroup$
                  That means $lim_{nrightarrowinfty}frac{n^b}{a^n}=lim_{nrightarrowinfty}frac{(n+1)^b}{a^(n+1)}$, so it must be zero. Is my understanding right?
                  $endgroup$
                  – ablmf
                  Aug 4 '11 at 3:18










                • $begingroup$
                  Yes, that's one good way to understand it.
                  $endgroup$
                  – Gerry Myerson
                  Aug 4 '11 at 3:50










                • $begingroup$
                  @GerryMyerson I’m just reading this right now, but that’s a bad way to understand it: It isn’t clear yet that the limit exists at all and for any converging sequence $lim a_n = lim a_{n+1}$ holds, so one cannot deduce anything about the limit from this or am I missing something? Maybe ablmf meant $lim_{n → ∞} {n^b}/{a^n} = lim_{n → ∞} {n^b}/{a^{n+1}}$? But here again you first need to know that the limit exists.
                  $endgroup$
                  – k.stm
                  Jun 2 '13 at 17:55










                3




                3




                $begingroup$
                This is the simplest solution I can think of.
                $endgroup$
                – Qiaochu Yuan
                Aug 4 '11 at 2:59




                $begingroup$
                This is the simplest solution I can think of.
                $endgroup$
                – Qiaochu Yuan
                Aug 4 '11 at 2:59




                1




                1




                $begingroup$
                This is a special case of multiplicative telescopy. See my post here for much further discussion of this example. Similar ideas work more generally for hyperrational functions.
                $endgroup$
                – Bill Dubuque
                Aug 4 '11 at 3:02






                $begingroup$
                This is a special case of multiplicative telescopy. See my post here for much further discussion of this example. Similar ideas work more generally for hyperrational functions.
                $endgroup$
                – Bill Dubuque
                Aug 4 '11 at 3:02














                $begingroup$
                That means $lim_{nrightarrowinfty}frac{n^b}{a^n}=lim_{nrightarrowinfty}frac{(n+1)^b}{a^(n+1)}$, so it must be zero. Is my understanding right?
                $endgroup$
                – ablmf
                Aug 4 '11 at 3:18




                $begingroup$
                That means $lim_{nrightarrowinfty}frac{n^b}{a^n}=lim_{nrightarrowinfty}frac{(n+1)^b}{a^(n+1)}$, so it must be zero. Is my understanding right?
                $endgroup$
                – ablmf
                Aug 4 '11 at 3:18












                $begingroup$
                Yes, that's one good way to understand it.
                $endgroup$
                – Gerry Myerson
                Aug 4 '11 at 3:50




                $begingroup$
                Yes, that's one good way to understand it.
                $endgroup$
                – Gerry Myerson
                Aug 4 '11 at 3:50












                $begingroup$
                @GerryMyerson I’m just reading this right now, but that’s a bad way to understand it: It isn’t clear yet that the limit exists at all and for any converging sequence $lim a_n = lim a_{n+1}$ holds, so one cannot deduce anything about the limit from this or am I missing something? Maybe ablmf meant $lim_{n → ∞} {n^b}/{a^n} = lim_{n → ∞} {n^b}/{a^{n+1}}$? But here again you first need to know that the limit exists.
                $endgroup$
                – k.stm
                Jun 2 '13 at 17:55






                $begingroup$
                @GerryMyerson I’m just reading this right now, but that’s a bad way to understand it: It isn’t clear yet that the limit exists at all and for any converging sequence $lim a_n = lim a_{n+1}$ holds, so one cannot deduce anything about the limit from this or am I missing something? Maybe ablmf meant $lim_{n → ∞} {n^b}/{a^n} = lim_{n → ∞} {n^b}/{a^{n+1}}$? But here again you first need to know that the limit exists.
                $endgroup$
                – k.stm
                Jun 2 '13 at 17:55













                9












                $begingroup$

                You can expand as$$a^n = e^{nlog a} = 1+nlog a +cdots +frac{(nlog a)^b}{b!}+cdots$$



                $$frac{a^n}{n^b} =frac{1}{n^b}+frac{1}{n^{b-1}}log a+cdots+frac{(log a)^{b}}{b!}+nfrac{(log a)^{b+1}}{(b+1)!}+cdots $$
                So, $$frac{a^n}{n^b}geq nfrac{(log a)^{b+1}}{(b+1)!}$$. This becomes arbitrarily large with large $n$



                Therefore, $$lim_{nrightarrowinfty}frac{a^n}{n^b} = infty$$ Or $$lim_{nrightarrowinfty}frac{n^b}{a^n} = 0$$






                share|cite|improve this answer











                $endgroup$


















                  9












                  $begingroup$

                  You can expand as$$a^n = e^{nlog a} = 1+nlog a +cdots +frac{(nlog a)^b}{b!}+cdots$$



                  $$frac{a^n}{n^b} =frac{1}{n^b}+frac{1}{n^{b-1}}log a+cdots+frac{(log a)^{b}}{b!}+nfrac{(log a)^{b+1}}{(b+1)!}+cdots $$
                  So, $$frac{a^n}{n^b}geq nfrac{(log a)^{b+1}}{(b+1)!}$$. This becomes arbitrarily large with large $n$



                  Therefore, $$lim_{nrightarrowinfty}frac{a^n}{n^b} = infty$$ Or $$lim_{nrightarrowinfty}frac{n^b}{a^n} = 0$$






                  share|cite|improve this answer











                  $endgroup$
















                    9












                    9








                    9





                    $begingroup$

                    You can expand as$$a^n = e^{nlog a} = 1+nlog a +cdots +frac{(nlog a)^b}{b!}+cdots$$



                    $$frac{a^n}{n^b} =frac{1}{n^b}+frac{1}{n^{b-1}}log a+cdots+frac{(log a)^{b}}{b!}+nfrac{(log a)^{b+1}}{(b+1)!}+cdots $$
                    So, $$frac{a^n}{n^b}geq nfrac{(log a)^{b+1}}{(b+1)!}$$. This becomes arbitrarily large with large $n$



                    Therefore, $$lim_{nrightarrowinfty}frac{a^n}{n^b} = infty$$ Or $$lim_{nrightarrowinfty}frac{n^b}{a^n} = 0$$






                    share|cite|improve this answer











                    $endgroup$



                    You can expand as$$a^n = e^{nlog a} = 1+nlog a +cdots +frac{(nlog a)^b}{b!}+cdots$$



                    $$frac{a^n}{n^b} =frac{1}{n^b}+frac{1}{n^{b-1}}log a+cdots+frac{(log a)^{b}}{b!}+nfrac{(log a)^{b+1}}{(b+1)!}+cdots $$
                    So, $$frac{a^n}{n^b}geq nfrac{(log a)^{b+1}}{(b+1)!}$$. This becomes arbitrarily large with large $n$



                    Therefore, $$lim_{nrightarrowinfty}frac{a^n}{n^b} = infty$$ Or $$lim_{nrightarrowinfty}frac{n^b}{a^n} = 0$$







                    share|cite|improve this answer














                    share|cite|improve this answer



                    share|cite|improve this answer








                    edited Aug 4 '11 at 4:00

























                    answered Aug 4 '11 at 3:54









                    kuch nahikuch nahi

                    3,44853267




                    3,44853267























                        3












                        $begingroup$

                        Regard the exponential function as the unique solution to the initial value problem $f(0) = 1$ with $f' = f$. The equivalent integral formulation is



                        $f(t) = 1 + int_0^t f(s) ds$



                        First prove that $f$ is an increasing function. For $t geq 0$, you can use a method of continuity argument by considering the set ${ s ~:~ f mbox{ increases on } [0, s] }$. The supremum of this set cannot be finite. For $s < 0$ you can also consider the ODE satisfied by $f^2$, but you're not really concerned with that. Now that we know $f$ increase, we also know $f$ grows at least linearly:



                        $f(t) geq 1 + int_0^t 1 ds = (1 + t)$



                        It also grows at least quadratically..



                        $f(t) geq f(frac{t}{2}) + int_{t/2}^t f(s) ds geq (1 + t/2) f(t/2)$



                        And repeating yields



                        $f(t) geq (1 + t/2)^2$



                        In general, $f(t) geq (1 + t/n)^n$ by the same method. It's even true for $n$ not an integer. In this case, the function $(1 + t/n)^n$ is defined in the first place to be $e^{n log( 1 + t/n) }$. Then, since log x is defined by the integral $int_1^x frac{1}{t} dt = x int_0^1 (1 + sx)^{-1} ds $, we can rewrite



                        $(1 + t/n)^n = e^{n log (1 + t/n) } = mbox{exp}(t int_0^1 (1 + frac{s t}{n})^{-1} ds ) leq e^t$



                        EDIT: This answer may not be so helpful to you. I didn't read the end of your question, so you may not know how to work with most of the things I've been writing about. The main reason why the exponential grows faster than a polynomial is because if $f$ is exponential, then $f(n+1)$ is at least a constant times $f(n)$, whereas when $f$ is a polynomial, $f(n+1)$ is roughly the same size as $f(n)$ when $n$ is large. After all, you can hardly tell the difference between the volume of a huge cube, and a huge cube 1 unit longer in each direction.






                        share|cite|improve this answer









                        $endgroup$













                        • $begingroup$
                          Oops, seems that I won't be able to understand this before I finish my calculus course.
                          $endgroup$
                          – ablmf
                          Aug 4 '11 at 2:59
















                        3












                        $begingroup$

                        Regard the exponential function as the unique solution to the initial value problem $f(0) = 1$ with $f' = f$. The equivalent integral formulation is



                        $f(t) = 1 + int_0^t f(s) ds$



                        First prove that $f$ is an increasing function. For $t geq 0$, you can use a method of continuity argument by considering the set ${ s ~:~ f mbox{ increases on } [0, s] }$. The supremum of this set cannot be finite. For $s < 0$ you can also consider the ODE satisfied by $f^2$, but you're not really concerned with that. Now that we know $f$ increase, we also know $f$ grows at least linearly:



                        $f(t) geq 1 + int_0^t 1 ds = (1 + t)$



                        It also grows at least quadratically..



                        $f(t) geq f(frac{t}{2}) + int_{t/2}^t f(s) ds geq (1 + t/2) f(t/2)$



                        And repeating yields



                        $f(t) geq (1 + t/2)^2$



                        In general, $f(t) geq (1 + t/n)^n$ by the same method. It's even true for $n$ not an integer. In this case, the function $(1 + t/n)^n$ is defined in the first place to be $e^{n log( 1 + t/n) }$. Then, since log x is defined by the integral $int_1^x frac{1}{t} dt = x int_0^1 (1 + sx)^{-1} ds $, we can rewrite



                        $(1 + t/n)^n = e^{n log (1 + t/n) } = mbox{exp}(t int_0^1 (1 + frac{s t}{n})^{-1} ds ) leq e^t$



                        EDIT: This answer may not be so helpful to you. I didn't read the end of your question, so you may not know how to work with most of the things I've been writing about. The main reason why the exponential grows faster than a polynomial is because if $f$ is exponential, then $f(n+1)$ is at least a constant times $f(n)$, whereas when $f$ is a polynomial, $f(n+1)$ is roughly the same size as $f(n)$ when $n$ is large. After all, you can hardly tell the difference between the volume of a huge cube, and a huge cube 1 unit longer in each direction.






                        share|cite|improve this answer









                        $endgroup$













                        • $begingroup$
                          Oops, seems that I won't be able to understand this before I finish my calculus course.
                          $endgroup$
                          – ablmf
                          Aug 4 '11 at 2:59














                        3












                        3








                        3





                        $begingroup$

                        Regard the exponential function as the unique solution to the initial value problem $f(0) = 1$ with $f' = f$. The equivalent integral formulation is



                        $f(t) = 1 + int_0^t f(s) ds$



                        First prove that $f$ is an increasing function. For $t geq 0$, you can use a method of continuity argument by considering the set ${ s ~:~ f mbox{ increases on } [0, s] }$. The supremum of this set cannot be finite. For $s < 0$ you can also consider the ODE satisfied by $f^2$, but you're not really concerned with that. Now that we know $f$ increase, we also know $f$ grows at least linearly:



                        $f(t) geq 1 + int_0^t 1 ds = (1 + t)$



                        It also grows at least quadratically..



                        $f(t) geq f(frac{t}{2}) + int_{t/2}^t f(s) ds geq (1 + t/2) f(t/2)$



                        And repeating yields



                        $f(t) geq (1 + t/2)^2$



                        In general, $f(t) geq (1 + t/n)^n$ by the same method. It's even true for $n$ not an integer. In this case, the function $(1 + t/n)^n$ is defined in the first place to be $e^{n log( 1 + t/n) }$. Then, since log x is defined by the integral $int_1^x frac{1}{t} dt = x int_0^1 (1 + sx)^{-1} ds $, we can rewrite



                        $(1 + t/n)^n = e^{n log (1 + t/n) } = mbox{exp}(t int_0^1 (1 + frac{s t}{n})^{-1} ds ) leq e^t$



                        EDIT: This answer may not be so helpful to you. I didn't read the end of your question, so you may not know how to work with most of the things I've been writing about. The main reason why the exponential grows faster than a polynomial is because if $f$ is exponential, then $f(n+1)$ is at least a constant times $f(n)$, whereas when $f$ is a polynomial, $f(n+1)$ is roughly the same size as $f(n)$ when $n$ is large. After all, you can hardly tell the difference between the volume of a huge cube, and a huge cube 1 unit longer in each direction.






                        share|cite|improve this answer









                        $endgroup$



                        Regard the exponential function as the unique solution to the initial value problem $f(0) = 1$ with $f' = f$. The equivalent integral formulation is



                        $f(t) = 1 + int_0^t f(s) ds$



                        First prove that $f$ is an increasing function. For $t geq 0$, you can use a method of continuity argument by considering the set ${ s ~:~ f mbox{ increases on } [0, s] }$. The supremum of this set cannot be finite. For $s < 0$ you can also consider the ODE satisfied by $f^2$, but you're not really concerned with that. Now that we know $f$ increase, we also know $f$ grows at least linearly:



                        $f(t) geq 1 + int_0^t 1 ds = (1 + t)$



                        It also grows at least quadratically..



                        $f(t) geq f(frac{t}{2}) + int_{t/2}^t f(s) ds geq (1 + t/2) f(t/2)$



                        And repeating yields



                        $f(t) geq (1 + t/2)^2$



                        In general, $f(t) geq (1 + t/n)^n$ by the same method. It's even true for $n$ not an integer. In this case, the function $(1 + t/n)^n$ is defined in the first place to be $e^{n log( 1 + t/n) }$. Then, since log x is defined by the integral $int_1^x frac{1}{t} dt = x int_0^1 (1 + sx)^{-1} ds $, we can rewrite



                        $(1 + t/n)^n = e^{n log (1 + t/n) } = mbox{exp}(t int_0^1 (1 + frac{s t}{n})^{-1} ds ) leq e^t$



                        EDIT: This answer may not be so helpful to you. I didn't read the end of your question, so you may not know how to work with most of the things I've been writing about. The main reason why the exponential grows faster than a polynomial is because if $f$ is exponential, then $f(n+1)$ is at least a constant times $f(n)$, whereas when $f$ is a polynomial, $f(n+1)$ is roughly the same size as $f(n)$ when $n$ is large. After all, you can hardly tell the difference between the volume of a huge cube, and a huge cube 1 unit longer in each direction.







                        share|cite|improve this answer












                        share|cite|improve this answer



                        share|cite|improve this answer










                        answered Aug 4 '11 at 2:13









                        user14222user14222

                        1612




                        1612












                        • $begingroup$
                          Oops, seems that I won't be able to understand this before I finish my calculus course.
                          $endgroup$
                          – ablmf
                          Aug 4 '11 at 2:59


















                        • $begingroup$
                          Oops, seems that I won't be able to understand this before I finish my calculus course.
                          $endgroup$
                          – ablmf
                          Aug 4 '11 at 2:59
















                        $begingroup$
                        Oops, seems that I won't be able to understand this before I finish my calculus course.
                        $endgroup$
                        – ablmf
                        Aug 4 '11 at 2:59




                        $begingroup$
                        Oops, seems that I won't be able to understand this before I finish my calculus course.
                        $endgroup$
                        – ablmf
                        Aug 4 '11 at 2:59











                        3












                        $begingroup$

                        Try expressing both in terms of the exponential $e^x$: $n^b$=$e^{bln(n)}$; $a^n=e^{nln(a)}$, so that the quotient becomes:



                        $frac{n^b}{a^n}$= $frac{e^{bln(n)}}{e^{nln(a)}}=e^{bln(n)-nln(a)}=e^{bln(n)-kn}=frac{e^{nlna}}{e^{kn}}$ , where k is a real constant. Can you tell which of ln(n) or n grows faster?






                        share|cite|improve this answer











                        $endgroup$













                        • $begingroup$
                          So the question becomes to prove $lim_{nrightarrowinfty}(bln(n)-kn) = -infty$ right? BTW, you made some typos.
                          $endgroup$
                          – ablmf
                          Aug 4 '11 at 2:58












                        • $begingroup$
                          Yes; I think too, if you use that ln(n) grows slower than n. I will check for typos.
                          $endgroup$
                          – gary
                          Aug 4 '11 at 3:46


















                        3












                        $begingroup$

                        Try expressing both in terms of the exponential $e^x$: $n^b$=$e^{bln(n)}$; $a^n=e^{nln(a)}$, so that the quotient becomes:



                        $frac{n^b}{a^n}$= $frac{e^{bln(n)}}{e^{nln(a)}}=e^{bln(n)-nln(a)}=e^{bln(n)-kn}=frac{e^{nlna}}{e^{kn}}$ , where k is a real constant. Can you tell which of ln(n) or n grows faster?






                        share|cite|improve this answer











                        $endgroup$













                        • $begingroup$
                          So the question becomes to prove $lim_{nrightarrowinfty}(bln(n)-kn) = -infty$ right? BTW, you made some typos.
                          $endgroup$
                          – ablmf
                          Aug 4 '11 at 2:58












                        • $begingroup$
                          Yes; I think too, if you use that ln(n) grows slower than n. I will check for typos.
                          $endgroup$
                          – gary
                          Aug 4 '11 at 3:46
















                        3












                        3








                        3





                        $begingroup$

                        Try expressing both in terms of the exponential $e^x$: $n^b$=$e^{bln(n)}$; $a^n=e^{nln(a)}$, so that the quotient becomes:



                        $frac{n^b}{a^n}$= $frac{e^{bln(n)}}{e^{nln(a)}}=e^{bln(n)-nln(a)}=e^{bln(n)-kn}=frac{e^{nlna}}{e^{kn}}$ , where k is a real constant. Can you tell which of ln(n) or n grows faster?






                        share|cite|improve this answer











                        $endgroup$



                        Try expressing both in terms of the exponential $e^x$: $n^b$=$e^{bln(n)}$; $a^n=e^{nln(a)}$, so that the quotient becomes:



                        $frac{n^b}{a^n}$= $frac{e^{bln(n)}}{e^{nln(a)}}=e^{bln(n)-nln(a)}=e^{bln(n)-kn}=frac{e^{nlna}}{e^{kn}}$ , where k is a real constant. Can you tell which of ln(n) or n grows faster?







                        share|cite|improve this answer














                        share|cite|improve this answer



                        share|cite|improve this answer








                        edited Aug 4 '11 at 3:48

























                        answered Aug 4 '11 at 1:52









                        garygary

                        3,49511119




                        3,49511119












                        • $begingroup$
                          So the question becomes to prove $lim_{nrightarrowinfty}(bln(n)-kn) = -infty$ right? BTW, you made some typos.
                          $endgroup$
                          – ablmf
                          Aug 4 '11 at 2:58












                        • $begingroup$
                          Yes; I think too, if you use that ln(n) grows slower than n. I will check for typos.
                          $endgroup$
                          – gary
                          Aug 4 '11 at 3:46




















                        • $begingroup$
                          So the question becomes to prove $lim_{nrightarrowinfty}(bln(n)-kn) = -infty$ right? BTW, you made some typos.
                          $endgroup$
                          – ablmf
                          Aug 4 '11 at 2:58












                        • $begingroup$
                          Yes; I think too, if you use that ln(n) grows slower than n. I will check for typos.
                          $endgroup$
                          – gary
                          Aug 4 '11 at 3:46


















                        $begingroup$
                        So the question becomes to prove $lim_{nrightarrowinfty}(bln(n)-kn) = -infty$ right? BTW, you made some typos.
                        $endgroup$
                        – ablmf
                        Aug 4 '11 at 2:58






                        $begingroup$
                        So the question becomes to prove $lim_{nrightarrowinfty}(bln(n)-kn) = -infty$ right? BTW, you made some typos.
                        $endgroup$
                        – ablmf
                        Aug 4 '11 at 2:58














                        $begingroup$
                        Yes; I think too, if you use that ln(n) grows slower than n. I will check for typos.
                        $endgroup$
                        – gary
                        Aug 4 '11 at 3:46






                        $begingroup$
                        Yes; I think too, if you use that ln(n) grows slower than n. I will check for typos.
                        $endgroup$
                        – gary
                        Aug 4 '11 at 3:46













                        3












                        $begingroup$

                        This is inspired by André Nicolas's answer, but instead of taking $b^{th}$ roots, I'll take $n^{th}$ roots.



                        The limit of the sequence of $n^{th}$ roots of the terms in the original sequence is
                        $$lim_{ntoinfty}frac{(n^b)^{1/n}}{a}=lim_{ntoinfty}frac{left(n^{1/n}right)^b}{a}=frac{left(limlimits_{ntoinfty} n^{1/n}right)^b}{a}=frac{1^b}{a}=frac{1}{a}.$$



                        (I used here the fact that $limlimits_{ntoinfty}n^{1/n}=1$, which was the subject of another question on this site, and which can also be proved in many ways.)



                        If $c$ is any number such that $frac{1}{a}<c<1$, then the previous limit implies that there is an $N>0$ such that for all $n>N$, $$left(frac{n^b}{a^n}right)^{1/n}lt c.$$ Then for all $n>N$, $$frac{n^b}{a^n}lt c^n.$$ Since $c^nto 0$, this shows that the original series goes to zero.



                        Alternatively, once you know that the sequence of $n^{th}$ roots converges to a number less than $1$, you can apply the root test to conclude that $sumlimits_{n=1}^inftyfrac{n^b}{a^n}$ converges, which implies that the terms go to $0$. (The proof of the root test actually just uses the inequality derived above and the convergence of the geometric series with common ratio $c$.)






                        share|cite|improve this answer











                        $endgroup$


















                          3












                          $begingroup$

                          This is inspired by André Nicolas's answer, but instead of taking $b^{th}$ roots, I'll take $n^{th}$ roots.



                          The limit of the sequence of $n^{th}$ roots of the terms in the original sequence is
                          $$lim_{ntoinfty}frac{(n^b)^{1/n}}{a}=lim_{ntoinfty}frac{left(n^{1/n}right)^b}{a}=frac{left(limlimits_{ntoinfty} n^{1/n}right)^b}{a}=frac{1^b}{a}=frac{1}{a}.$$



                          (I used here the fact that $limlimits_{ntoinfty}n^{1/n}=1$, which was the subject of another question on this site, and which can also be proved in many ways.)



                          If $c$ is any number such that $frac{1}{a}<c<1$, then the previous limit implies that there is an $N>0$ such that for all $n>N$, $$left(frac{n^b}{a^n}right)^{1/n}lt c.$$ Then for all $n>N$, $$frac{n^b}{a^n}lt c^n.$$ Since $c^nto 0$, this shows that the original series goes to zero.



                          Alternatively, once you know that the sequence of $n^{th}$ roots converges to a number less than $1$, you can apply the root test to conclude that $sumlimits_{n=1}^inftyfrac{n^b}{a^n}$ converges, which implies that the terms go to $0$. (The proof of the root test actually just uses the inequality derived above and the convergence of the geometric series with common ratio $c$.)






                          share|cite|improve this answer











                          $endgroup$
















                            3












                            3








                            3





                            $begingroup$

                            This is inspired by André Nicolas's answer, but instead of taking $b^{th}$ roots, I'll take $n^{th}$ roots.



                            The limit of the sequence of $n^{th}$ roots of the terms in the original sequence is
                            $$lim_{ntoinfty}frac{(n^b)^{1/n}}{a}=lim_{ntoinfty}frac{left(n^{1/n}right)^b}{a}=frac{left(limlimits_{ntoinfty} n^{1/n}right)^b}{a}=frac{1^b}{a}=frac{1}{a}.$$



                            (I used here the fact that $limlimits_{ntoinfty}n^{1/n}=1$, which was the subject of another question on this site, and which can also be proved in many ways.)



                            If $c$ is any number such that $frac{1}{a}<c<1$, then the previous limit implies that there is an $N>0$ such that for all $n>N$, $$left(frac{n^b}{a^n}right)^{1/n}lt c.$$ Then for all $n>N$, $$frac{n^b}{a^n}lt c^n.$$ Since $c^nto 0$, this shows that the original series goes to zero.



                            Alternatively, once you know that the sequence of $n^{th}$ roots converges to a number less than $1$, you can apply the root test to conclude that $sumlimits_{n=1}^inftyfrac{n^b}{a^n}$ converges, which implies that the terms go to $0$. (The proof of the root test actually just uses the inequality derived above and the convergence of the geometric series with common ratio $c$.)






                            share|cite|improve this answer











                            $endgroup$



                            This is inspired by André Nicolas's answer, but instead of taking $b^{th}$ roots, I'll take $n^{th}$ roots.



                            The limit of the sequence of $n^{th}$ roots of the terms in the original sequence is
                            $$lim_{ntoinfty}frac{(n^b)^{1/n}}{a}=lim_{ntoinfty}frac{left(n^{1/n}right)^b}{a}=frac{left(limlimits_{ntoinfty} n^{1/n}right)^b}{a}=frac{1^b}{a}=frac{1}{a}.$$



                            (I used here the fact that $limlimits_{ntoinfty}n^{1/n}=1$, which was the subject of another question on this site, and which can also be proved in many ways.)



                            If $c$ is any number such that $frac{1}{a}<c<1$, then the previous limit implies that there is an $N>0$ such that for all $n>N$, $$left(frac{n^b}{a^n}right)^{1/n}lt c.$$ Then for all $n>N$, $$frac{n^b}{a^n}lt c^n.$$ Since $c^nto 0$, this shows that the original series goes to zero.



                            Alternatively, once you know that the sequence of $n^{th}$ roots converges to a number less than $1$, you can apply the root test to conclude that $sumlimits_{n=1}^inftyfrac{n^b}{a^n}$ converges, which implies that the terms go to $0$. (The proof of the root test actually just uses the inequality derived above and the convergence of the geometric series with common ratio $c$.)







                            share|cite|improve this answer














                            share|cite|improve this answer



                            share|cite|improve this answer








                            edited Apr 13 '17 at 12:20









                            Community

                            1




                            1










                            answered Aug 4 '11 at 8:36









                            Jonas MeyerJonas Meyer

                            41.2k6149260




                            41.2k6149260























                                3












                                $begingroup$

                                Since it is almost always better to expand around zero, let $a = 1+c$, where $c > 0$. So we want to show $lim frac{n^b}{(1+c)^n} = 0$.



                                The ratio of consecutive terms is $frac{(1+1/n)^b}{1+c}$, so if we can show that $(1+1/n)^b < 1+c/2$ for large enough n, we are done. But this means that
                                $n > 1/((1+c/2)^{1/b}-1)$.



                                Restating, letting $N = frac{1}{(1+c/2)^{1/b}-1}$ and $r = frac{1+c/2}{1+c}$, if $n > N$ then $frac{(1+1/n)^b}{1+c} < frac{1+c/2}{1+c}$, which shows that
                                $$ frac{n^b}{(1+c)^n} < frac{N^b}{(1+c)^N}r^{n-N}$$
                                which goes to $0$ as $n rightarrow infty$ since $N$ is fixed and $0 < r < 1$.



                                Note: An elementary proof that $r^n rightarrow 0$ is in "What is Mathematics?" by Courant and Robbins.



                                Let $r = 1/(1+s)$, where $s > 0$. By Bernoulli's inequality,
                                $$r^n = frac{1}{(1+s)^n} < frac{1}{1+s n} = frac{1}{1+ n (1/r-1)}.$$
                                They similarly show that, if $a > 1$, then $a^n rightarrow infty$ like this:
                                Let $ a = 1+b$, where $b > 0$. Then
                                $$a^n = (1+b)^n > 1+n b = 1+n(a-1).$$
                                The keys in both these are expanding around zero and using Bernoulli's inequality.
                                Archimede's axiom (for any positive reals $x$ and $y$, there is an integer $n$ such that $x < n y$) does the rest.






                                share|cite|improve this answer









                                $endgroup$


















                                  3












                                  $begingroup$

                                  Since it is almost always better to expand around zero, let $a = 1+c$, where $c > 0$. So we want to show $lim frac{n^b}{(1+c)^n} = 0$.



                                  The ratio of consecutive terms is $frac{(1+1/n)^b}{1+c}$, so if we can show that $(1+1/n)^b < 1+c/2$ for large enough n, we are done. But this means that
                                  $n > 1/((1+c/2)^{1/b}-1)$.



                                  Restating, letting $N = frac{1}{(1+c/2)^{1/b}-1}$ and $r = frac{1+c/2}{1+c}$, if $n > N$ then $frac{(1+1/n)^b}{1+c} < frac{1+c/2}{1+c}$, which shows that
                                  $$ frac{n^b}{(1+c)^n} < frac{N^b}{(1+c)^N}r^{n-N}$$
                                  which goes to $0$ as $n rightarrow infty$ since $N$ is fixed and $0 < r < 1$.



                                  Note: An elementary proof that $r^n rightarrow 0$ is in "What is Mathematics?" by Courant and Robbins.



                                  Let $r = 1/(1+s)$, where $s > 0$. By Bernoulli's inequality,
                                  $$r^n = frac{1}{(1+s)^n} < frac{1}{1+s n} = frac{1}{1+ n (1/r-1)}.$$
                                  They similarly show that, if $a > 1$, then $a^n rightarrow infty$ like this:
                                  Let $ a = 1+b$, where $b > 0$. Then
                                  $$a^n = (1+b)^n > 1+n b = 1+n(a-1).$$
                                  The keys in both these are expanding around zero and using Bernoulli's inequality.
                                  Archimede's axiom (for any positive reals $x$ and $y$, there is an integer $n$ such that $x < n y$) does the rest.






                                  share|cite|improve this answer









                                  $endgroup$
















                                    3












                                    3








                                    3





                                    $begingroup$

                                    Since it is almost always better to expand around zero, let $a = 1+c$, where $c > 0$. So we want to show $lim frac{n^b}{(1+c)^n} = 0$.



                                    The ratio of consecutive terms is $frac{(1+1/n)^b}{1+c}$, so if we can show that $(1+1/n)^b < 1+c/2$ for large enough n, we are done. But this means that
                                    $n > 1/((1+c/2)^{1/b}-1)$.



                                    Restating, letting $N = frac{1}{(1+c/2)^{1/b}-1}$ and $r = frac{1+c/2}{1+c}$, if $n > N$ then $frac{(1+1/n)^b}{1+c} < frac{1+c/2}{1+c}$, which shows that
                                    $$ frac{n^b}{(1+c)^n} < frac{N^b}{(1+c)^N}r^{n-N}$$
                                    which goes to $0$ as $n rightarrow infty$ since $N$ is fixed and $0 < r < 1$.



                                    Note: An elementary proof that $r^n rightarrow 0$ is in "What is Mathematics?" by Courant and Robbins.



                                    Let $r = 1/(1+s)$, where $s > 0$. By Bernoulli's inequality,
                                    $$r^n = frac{1}{(1+s)^n} < frac{1}{1+s n} = frac{1}{1+ n (1/r-1)}.$$
                                    They similarly show that, if $a > 1$, then $a^n rightarrow infty$ like this:
                                    Let $ a = 1+b$, where $b > 0$. Then
                                    $$a^n = (1+b)^n > 1+n b = 1+n(a-1).$$
                                    The keys in both these are expanding around zero and using Bernoulli's inequality.
                                    Archimede's axiom (for any positive reals $x$ and $y$, there is an integer $n$ such that $x < n y$) does the rest.






                                    share|cite|improve this answer









                                    $endgroup$



                                    Since it is almost always better to expand around zero, let $a = 1+c$, where $c > 0$. So we want to show $lim frac{n^b}{(1+c)^n} = 0$.



                                    The ratio of consecutive terms is $frac{(1+1/n)^b}{1+c}$, so if we can show that $(1+1/n)^b < 1+c/2$ for large enough n, we are done. But this means that
                                    $n > 1/((1+c/2)^{1/b}-1)$.



                                    Restating, letting $N = frac{1}{(1+c/2)^{1/b}-1}$ and $r = frac{1+c/2}{1+c}$, if $n > N$ then $frac{(1+1/n)^b}{1+c} < frac{1+c/2}{1+c}$, which shows that
                                    $$ frac{n^b}{(1+c)^n} < frac{N^b}{(1+c)^N}r^{n-N}$$
                                    which goes to $0$ as $n rightarrow infty$ since $N$ is fixed and $0 < r < 1$.



                                    Note: An elementary proof that $r^n rightarrow 0$ is in "What is Mathematics?" by Courant and Robbins.



                                    Let $r = 1/(1+s)$, where $s > 0$. By Bernoulli's inequality,
                                    $$r^n = frac{1}{(1+s)^n} < frac{1}{1+s n} = frac{1}{1+ n (1/r-1)}.$$
                                    They similarly show that, if $a > 1$, then $a^n rightarrow infty$ like this:
                                    Let $ a = 1+b$, where $b > 0$. Then
                                    $$a^n = (1+b)^n > 1+n b = 1+n(a-1).$$
                                    The keys in both these are expanding around zero and using Bernoulli's inequality.
                                    Archimede's axiom (for any positive reals $x$ and $y$, there is an integer $n$ such that $x < n y$) does the rest.







                                    share|cite|improve this answer












                                    share|cite|improve this answer



                                    share|cite|improve this answer










                                    answered Aug 8 '11 at 13:43









                                    marty cohenmarty cohen

                                    76k549130




                                    76k549130























                                        1












                                        $begingroup$

                                        Tom Apostol, in volume I of his 2-volume text on calculus, gives what I consider to be the canonical proof of this assertion. He calls it the theorem of “the slow growth of the logarithm”. By making the appropriate substitution (namely, exp(x) for x), you can get what we can call the theorem on “the fast growth of the exponential”.



                                        I think you will have no difficulty agreeing that as far as comparing a polynomial to the exponential goes, the only part of the polynomial that we have to consider is the term involving the highest power, and that we can even disregard its coefficient. (This agreement is currently shown in your question, but I believe that is the result of someone else’s edit, and so I am answering what I believe your original question was.) That is what that theorem in Apostol does: it shows that if a > 0 and b > 0, then the limit as x goes to infinity of ((log(x))^a)/(x^b) goes to 0.



                                        You might be interested is knowing a significant application of this fact, namely, in the proof of the recursion formula for what is known as the Gamma function (which is the canonical continuous extension of the factorial function but, for historical reasons, shifted one unit). That is, in proving that Gamma(x + 1) = xGamma(x), one typically applies the technique of integration-by-parts, and one piece of the resulting expression is a power divided by an exponential, which goes to 0 as one takes the limit as the upper limit of the integral goes to infinity.



                                        Another application of this fact is the shape of the graph of y = x*exp(x), namely, that as x goes to negative infinity, y goes to 0.






                                        share|cite|improve this answer











                                        $endgroup$


















                                          1












                                          $begingroup$

                                          Tom Apostol, in volume I of his 2-volume text on calculus, gives what I consider to be the canonical proof of this assertion. He calls it the theorem of “the slow growth of the logarithm”. By making the appropriate substitution (namely, exp(x) for x), you can get what we can call the theorem on “the fast growth of the exponential”.



                                          I think you will have no difficulty agreeing that as far as comparing a polynomial to the exponential goes, the only part of the polynomial that we have to consider is the term involving the highest power, and that we can even disregard its coefficient. (This agreement is currently shown in your question, but I believe that is the result of someone else’s edit, and so I am answering what I believe your original question was.) That is what that theorem in Apostol does: it shows that if a > 0 and b > 0, then the limit as x goes to infinity of ((log(x))^a)/(x^b) goes to 0.



                                          You might be interested is knowing a significant application of this fact, namely, in the proof of the recursion formula for what is known as the Gamma function (which is the canonical continuous extension of the factorial function but, for historical reasons, shifted one unit). That is, in proving that Gamma(x + 1) = xGamma(x), one typically applies the technique of integration-by-parts, and one piece of the resulting expression is a power divided by an exponential, which goes to 0 as one takes the limit as the upper limit of the integral goes to infinity.



                                          Another application of this fact is the shape of the graph of y = x*exp(x), namely, that as x goes to negative infinity, y goes to 0.






                                          share|cite|improve this answer











                                          $endgroup$
















                                            1












                                            1








                                            1





                                            $begingroup$

                                            Tom Apostol, in volume I of his 2-volume text on calculus, gives what I consider to be the canonical proof of this assertion. He calls it the theorem of “the slow growth of the logarithm”. By making the appropriate substitution (namely, exp(x) for x), you can get what we can call the theorem on “the fast growth of the exponential”.



                                            I think you will have no difficulty agreeing that as far as comparing a polynomial to the exponential goes, the only part of the polynomial that we have to consider is the term involving the highest power, and that we can even disregard its coefficient. (This agreement is currently shown in your question, but I believe that is the result of someone else’s edit, and so I am answering what I believe your original question was.) That is what that theorem in Apostol does: it shows that if a > 0 and b > 0, then the limit as x goes to infinity of ((log(x))^a)/(x^b) goes to 0.



                                            You might be interested is knowing a significant application of this fact, namely, in the proof of the recursion formula for what is known as the Gamma function (which is the canonical continuous extension of the factorial function but, for historical reasons, shifted one unit). That is, in proving that Gamma(x + 1) = xGamma(x), one typically applies the technique of integration-by-parts, and one piece of the resulting expression is a power divided by an exponential, which goes to 0 as one takes the limit as the upper limit of the integral goes to infinity.



                                            Another application of this fact is the shape of the graph of y = x*exp(x), namely, that as x goes to negative infinity, y goes to 0.






                                            share|cite|improve this answer











                                            $endgroup$



                                            Tom Apostol, in volume I of his 2-volume text on calculus, gives what I consider to be the canonical proof of this assertion. He calls it the theorem of “the slow growth of the logarithm”. By making the appropriate substitution (namely, exp(x) for x), you can get what we can call the theorem on “the fast growth of the exponential”.



                                            I think you will have no difficulty agreeing that as far as comparing a polynomial to the exponential goes, the only part of the polynomial that we have to consider is the term involving the highest power, and that we can even disregard its coefficient. (This agreement is currently shown in your question, but I believe that is the result of someone else’s edit, and so I am answering what I believe your original question was.) That is what that theorem in Apostol does: it shows that if a > 0 and b > 0, then the limit as x goes to infinity of ((log(x))^a)/(x^b) goes to 0.



                                            You might be interested is knowing a significant application of this fact, namely, in the proof of the recursion formula for what is known as the Gamma function (which is the canonical continuous extension of the factorial function but, for historical reasons, shifted one unit). That is, in proving that Gamma(x + 1) = xGamma(x), one typically applies the technique of integration-by-parts, and one piece of the resulting expression is a power divided by an exponential, which goes to 0 as one takes the limit as the upper limit of the integral goes to infinity.



                                            Another application of this fact is the shape of the graph of y = x*exp(x), namely, that as x goes to negative infinity, y goes to 0.







                                            share|cite|improve this answer














                                            share|cite|improve this answer



                                            share|cite|improve this answer








                                            edited Aug 7 '11 at 21:00

























                                            answered Aug 4 '11 at 3:55









                                            Mike JonesMike Jones

                                            2,58112738




                                            2,58112738























                                                1












                                                $begingroup$

                                                As several of the previous answers have noted it is enough to show that $lim_{xto infty} frac{x}{a^x}=0$. To prove this, in turn it is enough to prove that $lim_{x to infty} frac{a^x}{x} = infty$.



                                                Lets compute the derivative and second derivative of $f(x) = frac{a^x}{x}$. We find that
                                                $f'(x) = frac{a^x(x ln(a)-1)}{x^2}$ and $f''(x) = frac{a^x(x^2 ln^2(a)-2ln(a) x+2)}{x^3}$.



                                                We see that for large enough values of $x$, both $f'(x)$ and $f''(x)$ are positive. Thus, $f$ not only grows, it grows at an increasing rate. The only possible conclusion is that $lim_{xto infty} frac{a^x}{x} = infty$.



                                                With a little work this can be made into a completely rigorous proof.






                                                share|cite|improve this answer









                                                $endgroup$


















                                                  1












                                                  $begingroup$

                                                  As several of the previous answers have noted it is enough to show that $lim_{xto infty} frac{x}{a^x}=0$. To prove this, in turn it is enough to prove that $lim_{x to infty} frac{a^x}{x} = infty$.



                                                  Lets compute the derivative and second derivative of $f(x) = frac{a^x}{x}$. We find that
                                                  $f'(x) = frac{a^x(x ln(a)-1)}{x^2}$ and $f''(x) = frac{a^x(x^2 ln^2(a)-2ln(a) x+2)}{x^3}$.



                                                  We see that for large enough values of $x$, both $f'(x)$ and $f''(x)$ are positive. Thus, $f$ not only grows, it grows at an increasing rate. The only possible conclusion is that $lim_{xto infty} frac{a^x}{x} = infty$.



                                                  With a little work this can be made into a completely rigorous proof.






                                                  share|cite|improve this answer









                                                  $endgroup$
















                                                    1












                                                    1








                                                    1





                                                    $begingroup$

                                                    As several of the previous answers have noted it is enough to show that $lim_{xto infty} frac{x}{a^x}=0$. To prove this, in turn it is enough to prove that $lim_{x to infty} frac{a^x}{x} = infty$.



                                                    Lets compute the derivative and second derivative of $f(x) = frac{a^x}{x}$. We find that
                                                    $f'(x) = frac{a^x(x ln(a)-1)}{x^2}$ and $f''(x) = frac{a^x(x^2 ln^2(a)-2ln(a) x+2)}{x^3}$.



                                                    We see that for large enough values of $x$, both $f'(x)$ and $f''(x)$ are positive. Thus, $f$ not only grows, it grows at an increasing rate. The only possible conclusion is that $lim_{xto infty} frac{a^x}{x} = infty$.



                                                    With a little work this can be made into a completely rigorous proof.






                                                    share|cite|improve this answer









                                                    $endgroup$



                                                    As several of the previous answers have noted it is enough to show that $lim_{xto infty} frac{x}{a^x}=0$. To prove this, in turn it is enough to prove that $lim_{x to infty} frac{a^x}{x} = infty$.



                                                    Lets compute the derivative and second derivative of $f(x) = frac{a^x}{x}$. We find that
                                                    $f'(x) = frac{a^x(x ln(a)-1)}{x^2}$ and $f''(x) = frac{a^x(x^2 ln^2(a)-2ln(a) x+2)}{x^3}$.



                                                    We see that for large enough values of $x$, both $f'(x)$ and $f''(x)$ are positive. Thus, $f$ not only grows, it grows at an increasing rate. The only possible conclusion is that $lim_{xto infty} frac{a^x}{x} = infty$.



                                                    With a little work this can be made into a completely rigorous proof.







                                                    share|cite|improve this answer












                                                    share|cite|improve this answer



                                                    share|cite|improve this answer










                                                    answered Feb 18 '15 at 19:36









                                                    JohanJohan

                                                    1,2011821




                                                    1,2011821























                                                        1












                                                        $begingroup$

                                                        Another method is the following (this method was originally? used by W. Rudin in his PMA):



                                                        For $ngeqslant2(|lceil brceil|+1)$ (the absolute value of the ceiling of $b$) we have, using the binomial theorem:$$begin{aligned}a^n&=(1+(a-1))^n\\&>binom{n}{|lceil brceil|+1}(a-1)^{|lceil brceil|+1}\\&=dfrac{n(n-1)cdots(n-|lceil brceil|)}{(|lceil brceil|+1)!}(a-1)^{|lceil brceil|+1}\\&>dfrac{n^{|lceil brceil|+1}}{2^{|lceil brceil|+1}(|lceil brceil|+1)!}(a-1)^{|lceil brceil|+1}end{aligned}$$ so $$left(dfrac{2^{|lceil brceil|+1}(|lceil brceil|+1)!}{(a-1)^{|lceil brceil|+1}}right)cdotdfrac{1}{n}>dfrac{n^{|lceil brceil|}}{a^n}geqslantdfrac{n^b}{a^n}$$ and hence $$lim_{ntoinfty}dfrac{n^b}{a^n}=0.$$






                                                        share|cite|improve this answer









                                                        $endgroup$


















                                                          1












                                                          $begingroup$

                                                          Another method is the following (this method was originally? used by W. Rudin in his PMA):



                                                          For $ngeqslant2(|lceil brceil|+1)$ (the absolute value of the ceiling of $b$) we have, using the binomial theorem:$$begin{aligned}a^n&=(1+(a-1))^n\\&>binom{n}{|lceil brceil|+1}(a-1)^{|lceil brceil|+1}\\&=dfrac{n(n-1)cdots(n-|lceil brceil|)}{(|lceil brceil|+1)!}(a-1)^{|lceil brceil|+1}\\&>dfrac{n^{|lceil brceil|+1}}{2^{|lceil brceil|+1}(|lceil brceil|+1)!}(a-1)^{|lceil brceil|+1}end{aligned}$$ so $$left(dfrac{2^{|lceil brceil|+1}(|lceil brceil|+1)!}{(a-1)^{|lceil brceil|+1}}right)cdotdfrac{1}{n}>dfrac{n^{|lceil brceil|}}{a^n}geqslantdfrac{n^b}{a^n}$$ and hence $$lim_{ntoinfty}dfrac{n^b}{a^n}=0.$$






                                                          share|cite|improve this answer









                                                          $endgroup$
















                                                            1












                                                            1








                                                            1





                                                            $begingroup$

                                                            Another method is the following (this method was originally? used by W. Rudin in his PMA):



                                                            For $ngeqslant2(|lceil brceil|+1)$ (the absolute value of the ceiling of $b$) we have, using the binomial theorem:$$begin{aligned}a^n&=(1+(a-1))^n\\&>binom{n}{|lceil brceil|+1}(a-1)^{|lceil brceil|+1}\\&=dfrac{n(n-1)cdots(n-|lceil brceil|)}{(|lceil brceil|+1)!}(a-1)^{|lceil brceil|+1}\\&>dfrac{n^{|lceil brceil|+1}}{2^{|lceil brceil|+1}(|lceil brceil|+1)!}(a-1)^{|lceil brceil|+1}end{aligned}$$ so $$left(dfrac{2^{|lceil brceil|+1}(|lceil brceil|+1)!}{(a-1)^{|lceil brceil|+1}}right)cdotdfrac{1}{n}>dfrac{n^{|lceil brceil|}}{a^n}geqslantdfrac{n^b}{a^n}$$ and hence $$lim_{ntoinfty}dfrac{n^b}{a^n}=0.$$






                                                            share|cite|improve this answer









                                                            $endgroup$



                                                            Another method is the following (this method was originally? used by W. Rudin in his PMA):



                                                            For $ngeqslant2(|lceil brceil|+1)$ (the absolute value of the ceiling of $b$) we have, using the binomial theorem:$$begin{aligned}a^n&=(1+(a-1))^n\\&>binom{n}{|lceil brceil|+1}(a-1)^{|lceil brceil|+1}\\&=dfrac{n(n-1)cdots(n-|lceil brceil|)}{(|lceil brceil|+1)!}(a-1)^{|lceil brceil|+1}\\&>dfrac{n^{|lceil brceil|+1}}{2^{|lceil brceil|+1}(|lceil brceil|+1)!}(a-1)^{|lceil brceil|+1}end{aligned}$$ so $$left(dfrac{2^{|lceil brceil|+1}(|lceil brceil|+1)!}{(a-1)^{|lceil brceil|+1}}right)cdotdfrac{1}{n}>dfrac{n^{|lceil brceil|}}{a^n}geqslantdfrac{n^b}{a^n}$$ and hence $$lim_{ntoinfty}dfrac{n^b}{a^n}=0.$$







                                                            share|cite|improve this answer












                                                            share|cite|improve this answer



                                                            share|cite|improve this answer










                                                            answered Dec 7 '16 at 3:14









                                                            CIJCIJ

                                                            2,8281717




                                                            2,8281717























                                                                1












                                                                $begingroup$

                                                                The case $ ble 0$ is blatantly obvious since $n^b le 1$ for $ble 0$ and hence since $a>1$ we have, $$frac{n^b}{a^n} le frac{1}{a^n} =left(frac{1}{a}right)^nto 0$$



                                                                we assume $ b>0$ then,



                                                                $$frac{n^b}{a^n}=left(frac{b}{ln a} right)^bleft(nfrac{ln a}{b} e^{-nfrac{ln a}{b}}right)^b color{red}{overset{u= nfrac{ln a}{b}}{:=}left(frac{b}{ln a} right)^bleft(u e^{-u}right)^b} ~~~~ b>0,~~$$



                                                                Hence, $$lim_{ntoinfty}frac{n^b}{a^n}= lim_{utoinfty}left(frac{b}{ln a} right)^bleft(u e^{-u}right)^b=0$$









                                                                share|cite|improve this answer











                                                                $endgroup$


















                                                                  1












                                                                  $begingroup$

                                                                  The case $ ble 0$ is blatantly obvious since $n^b le 1$ for $ble 0$ and hence since $a>1$ we have, $$frac{n^b}{a^n} le frac{1}{a^n} =left(frac{1}{a}right)^nto 0$$



                                                                  we assume $ b>0$ then,



                                                                  $$frac{n^b}{a^n}=left(frac{b}{ln a} right)^bleft(nfrac{ln a}{b} e^{-nfrac{ln a}{b}}right)^b color{red}{overset{u= nfrac{ln a}{b}}{:=}left(frac{b}{ln a} right)^bleft(u e^{-u}right)^b} ~~~~ b>0,~~$$



                                                                  Hence, $$lim_{ntoinfty}frac{n^b}{a^n}= lim_{utoinfty}left(frac{b}{ln a} right)^bleft(u e^{-u}right)^b=0$$









                                                                  share|cite|improve this answer











                                                                  $endgroup$
















                                                                    1












                                                                    1








                                                                    1





                                                                    $begingroup$

                                                                    The case $ ble 0$ is blatantly obvious since $n^b le 1$ for $ble 0$ and hence since $a>1$ we have, $$frac{n^b}{a^n} le frac{1}{a^n} =left(frac{1}{a}right)^nto 0$$



                                                                    we assume $ b>0$ then,



                                                                    $$frac{n^b}{a^n}=left(frac{b}{ln a} right)^bleft(nfrac{ln a}{b} e^{-nfrac{ln a}{b}}right)^b color{red}{overset{u= nfrac{ln a}{b}}{:=}left(frac{b}{ln a} right)^bleft(u e^{-u}right)^b} ~~~~ b>0,~~$$



                                                                    Hence, $$lim_{ntoinfty}frac{n^b}{a^n}= lim_{utoinfty}left(frac{b}{ln a} right)^bleft(u e^{-u}right)^b=0$$









                                                                    share|cite|improve this answer











                                                                    $endgroup$



                                                                    The case $ ble 0$ is blatantly obvious since $n^b le 1$ for $ble 0$ and hence since $a>1$ we have, $$frac{n^b}{a^n} le frac{1}{a^n} =left(frac{1}{a}right)^nto 0$$



                                                                    we assume $ b>0$ then,



                                                                    $$frac{n^b}{a^n}=left(frac{b}{ln a} right)^bleft(nfrac{ln a}{b} e^{-nfrac{ln a}{b}}right)^b color{red}{overset{u= nfrac{ln a}{b}}{:=}left(frac{b}{ln a} right)^bleft(u e^{-u}right)^b} ~~~~ b>0,~~$$



                                                                    Hence, $$lim_{ntoinfty}frac{n^b}{a^n}= lim_{utoinfty}left(frac{b}{ln a} right)^bleft(u e^{-u}right)^b=0$$










                                                                    share|cite|improve this answer














                                                                    share|cite|improve this answer



                                                                    share|cite|improve this answer








                                                                    edited Jan 21 '18 at 19:51

























                                                                    answered Jan 1 '18 at 11:25









                                                                    Guy FsoneGuy Fsone

                                                                    17.3k43074




                                                                    17.3k43074























                                                                        1












                                                                        $begingroup$

                                                                        You could also see the following:
                                                                        $$lim_{nrightarrowinfty}frac{n^b}{a^n} = lim_{nrightarrowinfty}e^{log frac{n^b}{a^n}}$$



                                                                        Now $lim_{nrightarrowinfty}log (frac{n^b}{a^n})=lim_{nrightarrowinfty}(log (n^b) - log (a^n))=lim_{nrightarrowinfty} (blog(n)-nlog(a))=-infty$ assuming $a>1$.



                                                                        So $lim_{nrightarrowinfty}e^{log frac{n^b}{a^n}}=0=lim_{nrightarrowinfty}frac{n^b}{a^n}$



                                                                        Just to clarify the comment, I am adding the proof that $$lim_{nrightarrowinfty} (blog (n)-nlog (a))=-infty$$



                                                                        First by applying L'Hôpital's rule we get



                                                                        $$lim_{nrightarrowinfty} frac{n}{log(n)}=lim_{nrightarrowinfty} frac{1}{frac{1}{n}} to infty$$
                                                                        We have then
                                                                        $$lim_{nrightarrowinfty} (blog (n)-nlog (a))=lim_{nrightarrowinfty} log(n)(b-log(a)frac{n}{log(n)})=-infty$$






                                                                        share|cite|improve this answer











                                                                        $endgroup$













                                                                        • $begingroup$
                                                                          Then you need an upper bound of $log(n)$.
                                                                          $endgroup$
                                                                          – ablmf
                                                                          Apr 15 '18 at 9:18










                                                                        • $begingroup$
                                                                          I assumed obviously that is a known fact that $log (n) to infty$ as $n to infty$ and that you know how to prove $lim_{nrightarrowinfty} (blog n-nlog a)=-infty$ assuming $a>1$
                                                                          $endgroup$
                                                                          – Marco Bellocchi
                                                                          Apr 15 '18 at 10:03


















                                                                        1












                                                                        $begingroup$

                                                                        You could also see the following:
                                                                        $$lim_{nrightarrowinfty}frac{n^b}{a^n} = lim_{nrightarrowinfty}e^{log frac{n^b}{a^n}}$$



                                                                        Now $lim_{nrightarrowinfty}log (frac{n^b}{a^n})=lim_{nrightarrowinfty}(log (n^b) - log (a^n))=lim_{nrightarrowinfty} (blog(n)-nlog(a))=-infty$ assuming $a>1$.



                                                                        So $lim_{nrightarrowinfty}e^{log frac{n^b}{a^n}}=0=lim_{nrightarrowinfty}frac{n^b}{a^n}$



                                                                        Just to clarify the comment, I am adding the proof that $$lim_{nrightarrowinfty} (blog (n)-nlog (a))=-infty$$



                                                                        First by applying L'Hôpital's rule we get



                                                                        $$lim_{nrightarrowinfty} frac{n}{log(n)}=lim_{nrightarrowinfty} frac{1}{frac{1}{n}} to infty$$
                                                                        We have then
                                                                        $$lim_{nrightarrowinfty} (blog (n)-nlog (a))=lim_{nrightarrowinfty} log(n)(b-log(a)frac{n}{log(n)})=-infty$$






                                                                        share|cite|improve this answer











                                                                        $endgroup$













                                                                        • $begingroup$
                                                                          Then you need an upper bound of $log(n)$.
                                                                          $endgroup$
                                                                          – ablmf
                                                                          Apr 15 '18 at 9:18










                                                                        • $begingroup$
                                                                          I assumed obviously that is a known fact that $log (n) to infty$ as $n to infty$ and that you know how to prove $lim_{nrightarrowinfty} (blog n-nlog a)=-infty$ assuming $a>1$
                                                                          $endgroup$
                                                                          – Marco Bellocchi
                                                                          Apr 15 '18 at 10:03
















                                                                        1












                                                                        1








                                                                        1





                                                                        $begingroup$

                                                                        You could also see the following:
                                                                        $$lim_{nrightarrowinfty}frac{n^b}{a^n} = lim_{nrightarrowinfty}e^{log frac{n^b}{a^n}}$$



                                                                        Now $lim_{nrightarrowinfty}log (frac{n^b}{a^n})=lim_{nrightarrowinfty}(log (n^b) - log (a^n))=lim_{nrightarrowinfty} (blog(n)-nlog(a))=-infty$ assuming $a>1$.



                                                                        So $lim_{nrightarrowinfty}e^{log frac{n^b}{a^n}}=0=lim_{nrightarrowinfty}frac{n^b}{a^n}$



                                                                        Just to clarify the comment, I am adding the proof that $$lim_{nrightarrowinfty} (blog (n)-nlog (a))=-infty$$



                                                                        First by applying L'Hôpital's rule we get



                                                                        $$lim_{nrightarrowinfty} frac{n}{log(n)}=lim_{nrightarrowinfty} frac{1}{frac{1}{n}} to infty$$
                                                                        We have then
                                                                        $$lim_{nrightarrowinfty} (blog (n)-nlog (a))=lim_{nrightarrowinfty} log(n)(b-log(a)frac{n}{log(n)})=-infty$$






                                                                        share|cite|improve this answer











                                                                        $endgroup$



                                                                        You could also see the following:
                                                                        $$lim_{nrightarrowinfty}frac{n^b}{a^n} = lim_{nrightarrowinfty}e^{log frac{n^b}{a^n}}$$



                                                                        Now $lim_{nrightarrowinfty}log (frac{n^b}{a^n})=lim_{nrightarrowinfty}(log (n^b) - log (a^n))=lim_{nrightarrowinfty} (blog(n)-nlog(a))=-infty$ assuming $a>1$.



                                                                        So $lim_{nrightarrowinfty}e^{log frac{n^b}{a^n}}=0=lim_{nrightarrowinfty}frac{n^b}{a^n}$



                                                                        Just to clarify the comment, I am adding the proof that $$lim_{nrightarrowinfty} (blog (n)-nlog (a))=-infty$$



                                                                        First by applying L'Hôpital's rule we get



                                                                        $$lim_{nrightarrowinfty} frac{n}{log(n)}=lim_{nrightarrowinfty} frac{1}{frac{1}{n}} to infty$$
                                                                        We have then
                                                                        $$lim_{nrightarrowinfty} (blog (n)-nlog (a))=lim_{nrightarrowinfty} log(n)(b-log(a)frac{n}{log(n)})=-infty$$







                                                                        share|cite|improve this answer














                                                                        share|cite|improve this answer



                                                                        share|cite|improve this answer








                                                                        edited Jan 18 at 22:07









                                                                        kccu

                                                                        11.5k11231




                                                                        11.5k11231










                                                                        answered Apr 14 '18 at 13:26









                                                                        Marco BellocchiMarco Bellocchi

                                                                        5081412




                                                                        5081412












                                                                        • $begingroup$
                                                                          Then you need an upper bound of $log(n)$.
                                                                          $endgroup$
                                                                          – ablmf
                                                                          Apr 15 '18 at 9:18










                                                                        • $begingroup$
                                                                          I assumed obviously that is a known fact that $log (n) to infty$ as $n to infty$ and that you know how to prove $lim_{nrightarrowinfty} (blog n-nlog a)=-infty$ assuming $a>1$
                                                                          $endgroup$
                                                                          – Marco Bellocchi
                                                                          Apr 15 '18 at 10:03




















                                                                        • $begingroup$
                                                                          Then you need an upper bound of $log(n)$.
                                                                          $endgroup$
                                                                          – ablmf
                                                                          Apr 15 '18 at 9:18










                                                                        • $begingroup$
                                                                          I assumed obviously that is a known fact that $log (n) to infty$ as $n to infty$ and that you know how to prove $lim_{nrightarrowinfty} (blog n-nlog a)=-infty$ assuming $a>1$
                                                                          $endgroup$
                                                                          – Marco Bellocchi
                                                                          Apr 15 '18 at 10:03


















                                                                        $begingroup$
                                                                        Then you need an upper bound of $log(n)$.
                                                                        $endgroup$
                                                                        – ablmf
                                                                        Apr 15 '18 at 9:18




                                                                        $begingroup$
                                                                        Then you need an upper bound of $log(n)$.
                                                                        $endgroup$
                                                                        – ablmf
                                                                        Apr 15 '18 at 9:18












                                                                        $begingroup$
                                                                        I assumed obviously that is a known fact that $log (n) to infty$ as $n to infty$ and that you know how to prove $lim_{nrightarrowinfty} (blog n-nlog a)=-infty$ assuming $a>1$
                                                                        $endgroup$
                                                                        – Marco Bellocchi
                                                                        Apr 15 '18 at 10:03






                                                                        $begingroup$
                                                                        I assumed obviously that is a known fact that $log (n) to infty$ as $n to infty$ and that you know how to prove $lim_{nrightarrowinfty} (blog n-nlog a)=-infty$ assuming $a>1$
                                                                        $endgroup$
                                                                        – Marco Bellocchi
                                                                        Apr 15 '18 at 10:03




















                                                                        draft saved

                                                                        draft discarded




















































                                                                        Thanks for contributing an answer to Mathematics Stack Exchange!


                                                                        • Please be sure to answer the question. Provide details and share your research!

                                                                        But avoid



                                                                        • Asking for help, clarification, or responding to other answers.

                                                                        • Making statements based on opinion; back them up with references or personal experience.


                                                                        Use MathJax to format equations. MathJax reference.


                                                                        To learn more, see our tips on writing great answers.




                                                                        draft saved


                                                                        draft discarded














                                                                        StackExchange.ready(
                                                                        function () {
                                                                        StackExchange.openid.initPostLogin('.new-post-login', 'https%3a%2f%2fmath.stackexchange.com%2fquestions%2f55468%2fhow-to-prove-that-exponential-grows-faster-than-polynomial%23new-answer', 'question_page');
                                                                        }
                                                                        );

                                                                        Post as a guest















                                                                        Required, but never shown





















































                                                                        Required, but never shown














                                                                        Required, but never shown












                                                                        Required, but never shown







                                                                        Required, but never shown

































                                                                        Required, but never shown














                                                                        Required, but never shown












                                                                        Required, but never shown







                                                                        Required, but never shown







                                                                        Popular posts from this blog

                                                                        Human spaceflight

                                                                        Can not write log (Is /dev/pts mounted?) - openpty in Ubuntu-on-Windows?

                                                                        File:DeusFollowingSea.jpg